Крок 3 - Медицина 2012 осінь (буклет)

1 / 200
У хворого 42-х років раптово виник напад серцебиття, що супроводжувався загальною слабкістю. На ЕКГ: ритм правильний, ЧСС- 170/хв., QRS- 0,10 с. Після масажу в ділянці каротидного синуса зменшилась ЧСС до 75/хв. Яке порушення ритму було зареєстроване у хворого? A 42-year-old patient had a sudden attack of palpitations, which was accompanied by general weakness. On the ECG: the rhythm is correct, heart rate - 170/min., QRS - 0, 10 s. After the massage in the area of ​​the carotid sinus, the heart rate decreased to 75/min. What rhythm disturbance was registered in the patient?

Шлуночкова екстрасистолія Ventricular extrasystole

Надшлуночкова пароксизмальна тахікардія Supraventricular paroxysmal tachycardia

Пароксизм миготливої аритмії Atrial fibrillation paroxysm

Шлуночкова пароксизмальна тахікардія Ventricular paroxysmal tachycardia

Надшлуночкова екстрасистолiя Supraventricular extrasystole

2 / 200
Бригадою швидкої медичної допомоги доставлений хворий 35-ти років, у якого раптово з’явився різкий головний біль з нудотою та блюванням. Об’єктивно: Ps- 60/хв., напружений. АТ- 190/120 мм рт.ст., шкіра гіперемована, свідомість сплутана. Позитивні симптоми Керніга, Брудзинського з обох сторін. Ліквор кров’янистий, цитоз 220/3. Який з діагнозів найбільш імовірний? A 35-year-old patient, who suddenly developed a sharp headache with nausea and vomiting, was delivered by the ambulance team. Objectively: Ps- 60/ min., tense. Blood pressure - 190/120 mmHg, hyperemic skin, confused consciousness. Positive symptoms of Kernig, Brudzinsky on both sides. Bloody cerebrospinal fluid, cytosis 220/3. Which of the diagnoses is most probable?

Субарахноїдальний крововилив Subarachnoid hemorrhage

Гостра гіпертонічна енцефалопатія Acute hypertensive encephalopathy

Симпато-адреналовий криз Sympatho-adrenal crisis

Геморагічний інсульт Hemorrhagic stroke

Менінгококовий менінгіт Meningococcal meningitis

3 / 200
В хірургічне відділення доставлений хворий після ДТП. Попередній діагноз: розрив печінки, внутрішня кровотеча. Для уточнення діагнозу необхідно: A patient was brought to the surgical department after a traffic accident. Preliminary diagnosis: rupture of the liver, internal bleeding. To clarify the diagnosis, you need:

Колоноскопія Colonoscopy

Оглядова рентгенографія органів черевної порожнини Overview x-ray of abdominal organs

Лапароцентез Laparocentesis

Іригоскопія Irigoscopy

Фіброгастродуоденоскопія Fibrogastroduodenoscopy

4 / 200
Хворого з імплантованим електрокарді-остимулятором 'вдарило струмом'під час ремонту настільної лампи, після чого він протягом декількох хвилин був непритомний. Об’єктивно: ціаноз шкіри, АТ- 60/0 мм рт.ст., ЧСС- 25/хв. Найбільш імовірною причиною неефективності кровообігу є: A patient with an implanted pacemaker was 'shocked' while repairing a table lamp, after which he was unconscious for several minutes. Objectively: cyanosis of the skin, BP - 60/0 mm Hg, heart rate - 25/min. The most likely cause of inefficiency of blood circulation is:

Травматичний шок Traumatic shock

Ішемічний інсульт Ischemic stroke

Тромбоемболія легеневої артерії Thromboembolism of the pulmonary artery

Атріовентрикулярна блокада Atrioventricular block

Електромеханічна дисоціація Electromechanical dissociation

5 / 200
Хворий 42-х років доставлений у стаціонар в порядку екстреної допомоги: раптово виникло блювання малозміненою кров’ю. Пацієнт сповістив, що така кровотеча вже третя протягом останніх двох років. З анамнезу - 8 років тому хворів на вірусний гепатит ',' A 42-year-old patient was brought to the hospital as an emergency: he suddenly started vomiting with little changed blood. The patient reported that this is the third such bleeding in the last two years From the anamnesis - 8 years ago he was sick with viral hepatitis ','

Об’єктивно: на животі виявляється значна венозна сітка у вигляді 'голови медузи', пальпується збільшена селезінка і щільний край печінки на рівні реберної дуги. Ps 1 І5/хв. АТ- 100/50 мм рт.ст. Який найбільш імовірний діагноз? Objectively: a significant venous network in the form of a 'medusa head' is found on the abdomen, an enlarged spleen and a dense edge of the liver at the level of the costal arch are palpable. Ps 1 І5/ min. BP - 100/50 mm Hg. What is the most likely diagnosis?

Легенева кровотеча Pulmonary hemorrhage

Кровотеча з пухлини шлунка Bleeding from a stomach tumor

Кровотеча з варикозно змінених вен стравоходу 6- Синдром Меллорі-Вейса Bleeding from varicose veins of the esophagus 6- Mallory-Weiss syndrome

Гострокровоточива дуоденальна виразка Acute bleeding duodenal ulcer

6 / 200
У пацієнта 40-ка років, котрий впродовж 17-ти років хворіє на неспецифічний виразковий коліт, при останньому ендоскопічному дослідженні виявлено множинні запальні псевдополіпи у всіх відділах товстої кишки. Результати біопсії показали наявність в поліпах дисплазії ІІ-ІІІ ст. Яке лікування необхідно застосувати в даного хворого? In a 40-year-old patient who has been suffering from non-specific ulcerative colitis for 17 years, the last endoscopic examination revealed multiple inflammatory pseudopolyps in all sections of the colon. The results of the biopsy showed the presence of grade II-III dysplasia in the polyps. What treatment should be applied to this patient?

Рентгенотерапія X-ray therapy

Трансендоскопічне видалення поліпів Transendoscopic removal of polyps

Хіміотерапія Chemotherapy

Хірургічне Surgical

Медикаментозне Medical

7 / 200
Хвора 36-ти років доставлена до лікарні у важкому стані зі скаргами на утруднене дихання, напади ядухи, нападоподібний біль у животі, рідкі випорожнення, потовиділення, гіперсалівацію, дрібні посмикування м’язів. Напередодні обприскувала плодові дерева хлорофосом. Був встановлений діагноз: гостре отруєння хлорофосом. Який лабораторний показник найбільш інформативний для підтвердження діагнозу? A 36-year-old patient was brought to the hospital in serious condition with complaints of difficulty breathing, attacks of nausea, paroxysmal abdominal pain, loose stools, sweating, hypersalivation, small muscle twitching. She sprayed fruit trees with chlorophos the day before. The diagnosis was made: acute chlorophos poisoning. Which laboratory indicator is the most informative to confirm the diagnosis?

Зниження концентрації ацетилхоліну Decreased acetylcholine concentration

Анемія Anemia

Протеїнурія Proteinuria

Еозинофілія Eosinophilia

Зниження активності холінестерази Decreased cholinesterase activity

8 / 200
Робочий 48-ми років під час роботи з електроприладом зазнав електротравми. Об’єктивно: потерпілий непритомний, АТ-70/30 мм рт.ст., миготлива аритмія, виражена задишка, часом зупинка дихання. Укажіть першочергові дії лікаря: A 48-year-old worker was electrocuted while working with an electrical appliance. Objectively: the victim is unconscious, blood pressure 70/30 mm Hg, atrial fibrillation , pronounced shortness of breath, sometimes stopping breathing. Indicate the doctor's primary actions:

Створити підвищене положення нижніх кінцівок Create elevated position of lower limbs

Транспортування до лікувального закладу Transportation to a medical facility

Загальний масаж тіла General body massage

Серцево-легенева реанімація Cardiopulmonary resuscitation

Накладання стерильної пов’язки на місце ураження Applying a sterile dressing to the affected area

9 / 200
Хлопчик 5-ти років надійшов у дитяче відділення з бронхопневмонією. Страждає на атопічний дерматит. Після внутрі-шньом’язового введення ампіциліну появились відчуття стиснення у грудях, запаморочення, різка блідість, ціаноз, холодний піт, прискорене шумне дихання. Який із перелічених препаратів слід увести в маніпу-ляційній негайно? A 5-year-old boy was admitted to the children's department with bronchopneumonia. He suffers from atopic dermatitis. After the intramuscular injection of ampicillin, he developed chest tightness, dizziness, sharp pallor, cyanosis, cold sweat, accelerated noisy breathing. Which of the listed drugs should be administered in the manipulation room immediately?

Розчин адреналіну підшкірно Solution of adrenaline subcutaneously

Розчин тавегілу внутрішньовенно Tavegil intravenous solution

Розчин строфантину внутрішньовенно Strophanthine intravenous solution

Розчин преднізолону внутрішньовенно Prednisone intravenous solution

Розчин еуфіліну внутрішньовенно Euphilin intravenous solution

10 / 200
Хворий на цукровий діабет звечора прийняв 12 ОД інсуліну, не вечеряв. Надійшов у клініку з руховим збудженням в комі. АТ- 90/70 мм рт.ст., Ps- 108/хв. Шкірні покриви бліді, вологі, акроціаноз. Тони серця ритмічні, дихання поверхневе. Ht- 40%, глікемія - 2,8 ммоль/л. Яка лікувальна тактика? A patient with diabetes took 12 units of insulin in the evening, did not eat dinner. He came to the clinic with motor excitement in a coma. Blood pressure - 90/70 mm Hg, Ps- 108/min. The skin is pale, moist, acrocyanosis. Heart sounds are rhythmic, breathing is shallow. Ht- 40%, glycemia - 2.8 mmol/l. What are the treatment tactics?

5% розчин глюкози 5% glucose solution

40% розчин глюкози 40% glucose solution

10% розчин глюкози 10% glucose solution

0,9% розчин натрію хлориду 0.9% sodium chloride solution

1% мезатон 1% mesaton

11 / 200
Госпіталізована хвора 30-ти років з діагнозом: вагітність 25-26 тижнів, внутрішньоутробна загибель плоду, метро-ендометрит. Об’єктивно: ЧДР- 30/хв., Ps-124/хв., to- 39 oC, АТ- 80/60 мм рт.ст., ЦВТ-10 мм вод.ст. У крові: Hb- 90 г/л, ер.- 2, 7 • 1012/л, тромб.-120 • 109/л, лейк.-18, 6 • 109/л. Чим зумовлена важкість стану хворої? Hospitalized 30-year-old patient with a diagnosis: pregnancy 25-26 weeks, intrauterine death of the fetus, metro-endometritis. Objectively: CHDR - 30/min. , Ps-124/min., to- 39 oC, BP- 80/60 mm Hg, CVT-10 mm Hg In blood: Hb- 90 g/l, er.- 2, 7 • 1012 /l, thromb.-120 • 109/l, leuk.-18, 6 • 109/l. What is the cause of the severity of the patient's condition?

Геморагічний шок Hemorrhagic shock

Гіповолемічний шок Hypovolemic shock

Інфекційно-токсичний шок Infectious-toxic shock

Кардіогенний шок Cardiogenic shock

Анафілактичний шок Anaphylactic shock

12 / 200
Дитина впала з висоти. Скарги на біль у животі. Об’єктивно: стан середньої важкості. Болючість при перкусії живота, нестійке напруження м’язів. З чого слід почати обстеження? The child fell from a height. Complaints of pain in the abdomen. Objectively: the condition is of medium severity. Pain upon percussion of the abdomen, unstable muscle tension. What should be followed start examination?

Внутрішньовенна урографія Intravenous urography

УЗД Ultrasound

Рентгенографія черевної порожнини X-ray of the abdominal cavity

Лапароцентез Laparocentesis

Лапароскопія Laparoscopy

13 / 200
Хлопчик 7-ми років страждає на гемофілію А, що клінічно супроводжується гемартрозами колінних суглобів після незначних травм. Яка тактика в більшій мірі сприятиме поліпшенню прогнозу та перебігу гемартрозу? A 7-year-old boy suffers from hemophilia A, which is clinically accompanied by hemarthrosis of the knee joints after minor injuries. What tactics will improve the prognosis and course of hemarthrosis to a greater extent?

Введення фактору VIII відразу (навіть батьками на дому) при появі болю в суглобі Introduction of factor VIII immediately (even by parents at home) when joint pain occurs

Пункція суглобу з метою діагностики і лікування Puncture of the joint for the purpose of diagnosis and treatment

Рання іммобілізація та холод на суглоб. Призначення вітаміну К Early immobilization and cold on the joint. Prescribing vitamin K

Регулярне обстеження в гематологічному відділенні Regular examination in the hematology department

Іммобілізація суглоба та лікування НПЗЗ (індометацин, АСК) Joint immobilization and NSAID treatment (indomethacin, ASA)

14 / 200
Вагітна зі строком вагітності 36 тижнів прийшла у дитячу поліклініку на прийом до педіатра з дитиною 3-х років. Під час бесіди з лікарем жінка поскаржилась на нудоту, порушення зору. Об’єктивно: бліда, тахікардія, АТ-140/90 мм рт.ст, набряки нижніх кінцівок. Які дії лікаря-педіатра? A pregnant woman with a gestation period of 36 weeks came to the children's polyclinic for an appointment with a pediatrician with a 3-year-old child. During the conversation with the doctor, the woman complained of nausea, impaired vision Objectively: pale, tachycardia, blood pressure 140/90 mm Hg, edema of the lower extremities. What are the actions of the pediatrician?

Ввести гіпотензивні препарати, госпіталізація у пологовий будинок у супроводі лікаря Introduce antihypertensive drugs, hospitalization in the maternity hospital accompanied by a doctor

Порекомендувати призначення антибактеріальної терапії Recommend antibacterial therapy

Внутрішньовенно ввести 150 мг кокарбо-ксилази Enter 150 mg of cocarbo-xylase intravenously

Ввести спазмолітики, та при стабілізації артеріального тиску повернути додому Introduce antispasmodics, and return home when blood pressure stabilizes

Направити жінку на прийом до терапевта Send the woman to see a therapist

15 / 200
Дівчинка 6-ти років хворіє на бронхіальну астму з 3-х років, у неї відмічаються нічні напади ядухи, свистяче дихання. Під час останнього нападу ядухи тричі вдихала Сальбутамол - поліпшення стану у дівчинки не відмічалося. Які засоби лікування слід призначити? A 6-year-old girl has suffered from bronchial asthma since the age of 3, she has nocturnal attacks of wheezing, wheezing. During the last attack of wheezing, she inhaled Salbutamol three times - the girl's condition did not improve. What means of treatment should be prescribed?

Антибіотики Antibiotics

Дигоксин Digoxin

Преднізолон Prednisone

Мукалтин Mukaltin

Піпольфен Pipolfen

16 / 200
У дитини, що страждає на цукровий діабет, діагностовано гіперосмолярну кому. Глюкоза крові більше 50 ммоль/л, рівні NaCl та бікарбонатів підвищені, осмоляр-ність сироватки крові - 380 мосмоль/л. Які головні моменти в лікуванні гіперосмоляр-ної коми? A child suffering from diabetes is diagnosed with hyperosmolar coma. Blood glucose is more than 50 mmol/l, NaCl and bicarbonate levels are elevated, serum osmolarity - 380 mosmol/l. What are the main points in the treatment of hyperosmolar coma?

Внутрішньовенне введення 0,45% розчину NaCl, калію Intravenous administration of 0.45% NaCl solution, potassium

Внутрішньовенне введення гіпертонічного 40% розчину глюкози, калію Intravenous administration of hypertonic 40% solution of glucose, potassium

Внутрішньовенне введення гідрокарбонату натрію 4% розчину, калію Intravenous administration of sodium bicarbonate 4% solution, potassium

Внутрішньовенне введення 5% розчину глюкози, калію Intravenous administration of 5% glucose solution, potassium

Внутрішньовенне введення ізотонічного розчину NaCl, калію Intravenous administration of an isotonic solution of NaCl, potassium

17 / 200
Постраждалого вилучено з води після аварії транспортного судна. Він знаходиться в коматозному стані, температура тіла 25oC. Який захід першої медичної допомоги? The victim was pulled out of the water after the accident of the transport vessel. He is in a comatose state, the body temperature is 25oC. What is the first aid measure?

Зігрівання грілкою ділянки серця і печінки Warming of the heart and liver with a heating pad

Зігрівання потилиці Warming the back of the head

Зігрівання грудини, живота, шиї Warming the sternum, abdomen, neck

Промивання шлунка через зонд 5% розчином соди Gastric lavage through a probe with 5% soda solution

Поступове загальне зігрівання Gradual global warming

18 / 200
У хворого з інфекційним ендокардитом з ураженням аортального клапану виник інтенсивний біль у попереку, зліва та справа, який був розцінений як інфаркт нирок. Протягом доби у хворого посилились загальна слабкість, апатія, тахікардія, артеріальний тиск знизився до 60/0 мм рт.ст., з’явилися блювання, пронос, холодний липкий піт, олігурія. Чим ускладнився перебіг захворювання? A patient with infectious endocarditis with damage to the aortic valve developed intense pain in the lower back, left and right, which was regarded as a kidney infarction. Over the course of a day, the patient's general weakness worsened , apathy, tachycardia, blood pressure decreased to 60/0 mm Hg, vomiting, diarrhea, cold sticky sweat, oliguria appeared. What complicated the course of the disease?

Інфаркт мозку Cerebral infarction

Гостра надниркова недостатність внаслідок крововиливу у наднирники Acute adrenal insufficiency due to adrenal hemorrhage

Інфаркт селезінки Splenic infarction

Гостра ниркова недостатність внаслідок інфаркту нирки Acute renal failure due to renal infarction

Гостра серцева недостатність Acute heart failure

19 / 200
Пацієнтка 25-ти років за останній тиждень перед госпіталізацією стала злобливою, неспокійною, накидалася на матір, била її. Потім стала застигати у вигадливих позах, стереотипно викрикувала одне слово. За останню добу нерухома, не відповідає на запитання, відмовляється від їжі. Обличчя амімічне, щільно стиснуті щелепи, витягнуті вперед губи (хоботком), лежить у внутрішньоутробній позі, відзначається підвищення м’язового тонусу всього тіла. Будь-яка спроба змінити позу викликає різку напругу м’язів із протидією. Соматичний статус без особливостей, АТ-120/80 мм рт.ст., Ps- 75/хв. Неврологічний статус без ознак органічної патології ЦНС. Назвіть психопатологічний синдром: In the last week before hospitalization, a 25-year-old patient became angry, restless, lashed out at her mother, beat her. Then she began to freeze in strange poses, stereotypically shouted one word . For the last day, she has been motionless, does not answer questions, refuses to eat. The face is amemic, the jaws are tightly clenched, the lips (proboscis) are stretched forward, she is lying in the fetal position, there is an increase in the muscle tone of the whole body. Any attempt to change the position causes sharp muscle tension with counteraction. Somatic status without features, BP-120/80 mm Hg, Ps- 75/min. Neurological status without signs of organic CNS pathology. Name the psychopathological syndrome:

Параноїдний синдром Paranoid Syndrome

Галюцинаторно-параноїдний синдром Hallucinatory-paranoid syndrome

Кататонічний синдром Catatonic syndrome

Депресивний синдром Depressive syndrome

Маніакальний синдром Manic syndrome

20 / 200
Хворому 29-ти років проведена первинна хірургічна обробка розчавленої рани верхньої третини лівого стегна. Через 2 дні стан хворого погіршився. При огляді рана з незначними виділеннями, м’язи сіруватого кольору, кінцівка набрякла, шкіра холодна, вкрита синюшними плямами, при пальпації визначається крепітація. Свідомість запаморочена. Температура тіла 40,3°C. Який найбільш імовірний діагноз? A 29-year-old patient underwent primary surgical treatment of a crushed wound in the upper third of the left thigh. After 2 days, the patient's condition worsened. When examining the wound with minor discharge, muscles grayish in color, the limb is swollen, the skin is cold, covered with bluish spots, palpation reveals crepitus. Consciousness is dizzy. Body temperature is 40.3°C. What is the most likely diagnosis?

Газова гангрена Gas gangrene

Артеріальний тромбоз Arterial thrombosis

Правець Tetanus

Бешиха Beshikha

Флегмона стегна Phlegmon of the hip

21 / 200
Хворий 76-ти років надійшов у клініку зі скаргами на ниючий біль розпираючого характеру над лоном, відсутність сечі протягом 2-х діб. Об’єктивно: над лоном пальпується болюче утворення, верхній край якого сягає пупка. Передміхурова залоза під час пальпації через пряму кишку збільшена, щільної консистенції. Який найбільш імовірний діагноз? A 76-year-old patient came to the clinic with complaints of aching pain of a distending nature over the pubic area, no urine for 2 days. Objectively: palpable over the pubic area a painful formation, the upper edge of which reaches the navel. The prostate gland during palpation through the rectum is enlarged, with a dense consistency. What is the most likely diagnosis?

Сечокам’яна хвороба Urolithiasis

Пухлина сечового міхура Bladder tumor

Аденома передміхурової залози, гостра затримка сечі Adenoma of the prostate gland, acute urinary retention

Пухлина тонкого кишківника Tumor of the small intestine

Гостра кишкова непрохідність Acute intestinal obstruction

22 / 200
Чоловіку 33-х років під місцевою анестезією (1 мл 2% розчину лідокаїну) проведена екстракція зуба. Через 7 хвилин після знеболення і через 2 хвилини після видалення зуба хворий почав скаржитись на нестачу повітря, запаморочення, відчуття стиснення в грудній клітці. Об’єктивно: шкіра та слизові бліді, ЧДР- 32/хв., АТ-60/20 мм рт.ст., Ps112 /хв., слабкого наповнення. Чим зумовлено погіршення стану пацієнта? A 33-year-old man underwent tooth extraction under local anesthesia (1 ml of 2% lidocaine solution). 7 minutes after anesthesia and 2 minutes after tooth extraction, the patient began to complain of lack of air, dizziness, feeling of tightness in the chest. Objectively: skin and mucous membranes are pale, blood pressure - 32/min., blood pressure - 60/20 mmHg, Ps112 /min., weak filling. What caused by the deterioration of the patient's condition?

- -

Больовий шок Pain shock

Токсико-бактеріальний шок Toxic-bacterial shock

Кардіогенний шок Cardiogenic shock

Анафілактичний шок Anaphylactic shock

23 / 200
Хворий 48-ми років скаржиться на інтенсивний біль за грудниною, ядуху, кашель із виділенням пінистого харкотиння. Об’єктивно: стан важкий, акроціаноз, клекочуче дихання, Ps- 100/хв., АТ-180/110 мм рт.ст., серцеві тони приглушені. В нижніх відділах легень - вологі хрипи. ЕКГ: патологічний зубець Q в I, aVL, V1 — V6, зміщення ST на 4 мм вище ізолінії. Яке ускладнення розвинулось у хворого? A 48-year-old patient complains of intense chest pain, wheezing, cough with frothy sputum. Objectively: the condition is severe, acrocyanosis, wheezing, Ps - 100/min., BP-180/110 mmHg, heart sounds are muffled. In the lower parts of the lungs - moist rales. ECG: pathological Q wave in I, aVL, V1 — V6, ST shift 4 mm above the isoline What complications did the patient develop?

Пневмонія Pneumonia

Набряк легенів Pulmonary edema

Синдром Дреслера Dressler Syndrome

Тромбоемболія легеневої артерії Thromboembolism of the pulmonary artery

Кардіогенний шок Cardiogenic shock

24 / 200
Хворий 35-ти років, працівник котельні, госпіталізований із скаргами на інтенсивний головний біль, запаморочення, зорові галюцинації, шум у вухах, задишку в спокої, біль у грудній клітці, нудоту, значну м’язову слабість. Стан виник під час гасіння пожежі. Шкіра обличчя малинового кольору, анізокорія. АТ- 80/40 мм рт.ст., Ps-54/хв., t°- 35, 5°C. Ригідність м’язів потилиці. Тонічні судоми м’язів кінцівок. У крові - позитивна проба на карбоксигемоглобін. Який антидот слід застосувати? A 35-year-old patient, a boiler room worker, was hospitalized with complaints of intense headache, dizziness, visual hallucinations, tinnitus, shortness of breath at rest, chest pain cages, nausea, significant muscle weakness. The condition arose during firefighting. The skin of the face is crimson, anisocoria. BP- 80/40 mm Hg, Ps-54/min, t°- 35, 5°C Stiffness of the muscles of the back of the head. Tonic spasms of the muscles of the limbs. In the blood there is a positive test for carboxyhemoglobin. What antidote should be used?

Атропіну сульфат Atropine sulfate

Дипіроксим Dipiroxime

Пентацин Pentacin

Хромосмон Chromosmon

Унітіол Unithiol

25 / 200
Хворий 23-х років скаржиться на наявність набряків на обличчі, головний біль, появу темно-червоної сечі. Через декілька діб кількість сечі зменшилась до 300 мл/добу, виникла сухість у роті, спрага, біль у попереку. Об’єктивно: набряки на обличчі, температура - 39, 5°C; ЧСС- 80/хв., АТ- 170/110 мм рт.ст., систолічний шум на верхівці серця. Креатинін крові - 310 мкмоль/л, білок сєчі - 3,3 г/л, еритроцити вкривають все поле зору. Чим ускладнився перебіг захворювання у даного хворого? A 23-year-old patient complains of facial swelling, headache, dark red urine. After a few days, the amount of urine decreased to 300 ml/day, dry mouth, thirst, lower back pain appeared. Objectively: swelling on the face, temperature - 39.5°C, heart rate - 80/min, blood pressure - 170/110 mm Hg, systolic murmur at the top of the heart Blood creatinine - 310 μmol/l, urine protein - 3.3 g/l, erythrocytes cover the entire field of vision. What complicated the course of the disease in this patient?

Міокардитом Myocarditis

Приєднанням пієлонефриту Attachment of pyelonephritis

Уролітіазом Urolithiasis

Гіпертонічним кризом Hypertensive crisis

Нирковою недостатністю Kidney failure

26 / 200
- х річна жінка прокинувшись вранці відмітила, що в неї порушилася вимова, з’явилися поперхування під час ковтання, хиткість, зникла чутливість в лівій руці. Звернулася по допомогу через 2 доби. Об’єктивно: АТ- 135/85 мм рт.ст. Правобічний парез м’якого піднебіння, голосової зв’язки, міоз, птоз, енофтальм праворуч, гемігіпалгезія ліворуч. Який найбільш імовірний діагноз? - x-year-old woman woke up in the morning and noticed that her pronunciation was impaired, she had sputtering while swallowing, unsteadiness, sensitivity in her left hand disappeared. She asked for help after 2 days. Objectively: blood pressure - 135/85 mm Hg. Right-sided paresis of the soft palate, vocal cords, miosis, ptosis, enophthalmos on the right, hemihypalgesia on the left. What is the most likely diagnosis?

Пухлина головного мозку Brain tumor

Транзиторна ішемічна атака Transient ischemic attack

Ішемічний інсульт Ischemic stroke

Паренхіматозний крововилив Parenchymal hemorrhage

Субарахноїдальний крововилив Subarachnoid hemorrhage

27 / 200
Хворий 57-ми років 5 років страждає на стенокардію напруги, відмічає задишку, раптовий інтенсивний пекучий біль за грудниною, що триває 2,5 години. На ЕКГ: ритм синусовий, правильний, ЧСС-100/хв, у V3-5 сегмент ST на 8 мм вище ізолінії. Тони серця послаблені. АТ- 100/60 мм рт.ст. Які заходи невідкладної допомоги слід вжити першочергово? A 57-year-old patient has been suffering from tension angina for 5 years, notes shortness of breath, sudden intense burning pain behind the sternum lasting 2.5 hours. On the ECG: rhythm sinus, regular, heart rate - 100/min, in V3-5 ST segment 8 mm above the isoline. Heart sounds are weakened. BP - 100/60 mm Hg. What emergency measures should be taken as a priority?

Електрокардіостимуляція Electronic cardiostimulation

Інфузія гепарину Heparin infusion

Тромболітична терапія Thrombolytic therapy

Інфузія допаміну Dopamine infusion

Інгаляція кисню Oxygen inhalation

28 / 200
У хворої 48-ми років на фоні фіброміоми матки та метрорагій виникла анемія. Хворій рекомендовано оперативне лікування в зв’язку із метрорагією. У крові: ер.- 2,0 • 1012/л, НЬ- 50 г/л, КП- 0,75, тромб.-140-109/л,лейк.- 4,1 • 109/л,ШОЕ- 10мм/год. Які невідкладні заходи слід вжити для корекції анемії'? A 48-year-old patient developed anemia against the background of uterine fibroids and metrorrhagia. The patient was recommended surgical treatment in connection with metrorrhagia. In the blood: er.- 2 ,0 • 1012/l, Hb- 50 g/l, KP- 0.75, thromb.-140-109/l, leuk.- 4.1 • 109/l, ESR- 10 mm/h. What urgent measures should be taken take to correct anemia'?

Гемотрансфузія концентрату еритроцитів Hemotransfusion of red blood cell concentrate

Переливання цільної крові Whole blood transfusion

Вітамін B12 Vitamin B12

Парентеральні препарати заліза Parenteral iron preparations

Пероральні препарати заліза Oral iron preparations

29 / 200
У хворої 36-ти років впродовж останнього тижня неодноразово виникає лівостороння ниркова колька. Надійшла до урологічного відділення. Об’єктивно: пальпація лівої нирки болісна, симптом Па-стернацького позитивний. Спазмоаналге-тики біль зменшили незначно. За даними УЗД виражена дилатація сечоводу. Яка подальша тактика? A 36-year-old patient has repeatedly had left-sided renal colic over the past week. She was referred to the urology department. Objectively: palpation of the left kidney is painful, Pa-sternatsky's symptom positive. Antispasmodic analgesics reduced the pain slightly. According to the ultrasound, the ureter was dilated. What is the next strategy?

Введення диклофенаку внутрішньом’я-зово Intramuscular injection of diclofenac

Екстрена катетеризація сечоводу або черезшкірна пункційна нефростомія Emergency ureteral catheterization or percutaneous puncture nephrostomy

Новокаїнова блокада за Лорін-Епштейном Novocaine blockade according to Lorin-Epstein

Повторне введення лікарських засобів но-шпа, баралгін, промедол, платифілін та інше Re-introduction of drugs no-shpa, baralgin, promedol, platyfillin and others

Прийом тамсулозину (омніка) усередину Tamsulosin intake (Omnica) internally

30 / 200
Хворий 65-ти років доставлений до лікарні у непритомному стані. Зі слів дружини стан погіршився 3 дні тому: з’явилися лихоманка до 39°C, галюцинації, було сильне блювання. Об’єктивно: шкіра і слизові сухі, Ps- 110/хв., АТ- 100/70 мм рт.ст. ЧД- 24/хв. Печінка +2 см. У крові: ер.- 4,5 • 1012/л, НЬ- 155 г/л, лейк.-17,3 • 109/л, ШОЕ-18 мм/год. Глюкоза крові 50 ммоль/л, сечовина - 9 ммоль/л, осмо-лярність плазми - 380 мосмоль/л, білірубін загальний - 26,2 мкмоль/л. Який невідкладний стан розвинувся у пацієнта? A 65-year-old patient was brought to the hospital in an unconscious state. According to his wife, his condition worsened 3 days ago: fever up to 39°C appeared, hallucinations, severe vomiting. Objectively: dry skin and mucous membranes, Ps- 110/min., BP- 100/70 mm Hg, BH- 24/min. Liver +2 cm. In the blood: er.- 4.5 • 1012/l, Hb-155 g/l, leuk.-17.3 • 109/l, ESR-18 mm/h, blood glucose 50 mmol/l, urea - 9 mmol/l, plasma osmolality - 380 mosmol /l, total bilirubin - 26.2 μmol/l. What emergency condition has developed in the patient?

Уремічна кома Uremic coma

Гіперосмолярна кома Hyperosmolar coma

Кетоацидотична кома Ketoacidotic coma

Гіпоглікемічна кома Hypoglycemic coma

Печінкова кома Hepatic coma

31 / 200
Хворому 52-х років виконана екскреторна урографія. Через 10 хвилин після внутрішньовенного введення контрастної речовини стан хворого різко погіршився. Непритомний. АТ- 50/20 мм рт.ст., Ps-120/хв., ритмічний. В анамнезі - медикаментозна алергія. З метою лікування необхідно ввести: A 52-year-old patient underwent excretory urography. 10 minutes after the intravenous administration of contrast material, the patient's condition worsened sharply. He is unconscious. Blood pressure - 50/20 mm Hg ., Ps-120/min., rhythmic. There is a history of drug allergy. For the purpose of treatment, it is necessary to enter:

Адреналін підшкірно, димедрол внутрішньовенно Adrenaline subcutaneously, diphenhydramine intravenously

Внутрішньовенно преднізолон, інфузія кристалоїдів Intravenous prednisolone, crystalloid infusion

Внутрішньом’язово димедрол, преднізолон Intramuscular diphenhydramine, prednisolone

Внутрішньовенно еуфілін Intravenous euphilin

Адреналін внутрішньовенно, швидка інфузія кристалоїдів Adrenaline IV, rapid infusion of crystalloids

32 / 200
Хлопчика 7-ми років доставлено до стаціонару. Напередодні їв гриби. Анурія другу добу. У крові: сечовина - 26 ммоль/л, кре-атинін -1,2 ммоль/л, K - 7,8 ммоль/л, рН- 71, ВЕ- 11, Na110 ммоль/л. Який попередній діагноз? A 7-year-old boy was taken to the hospital. He ate mushrooms the day before. Anuria for the second day. In the blood: urea - 26 mmol/l, creatinine -1, 2 mmol/l, K - 7.8 mmol/l, pH - 71, VE - 11, Na110 mmol/l. What is the previous diagnosis?

Гостра затримка сечі Acute urinary retention

Гострий гломерулонефрит Acute glomerulonephritis

Отруєння грибами Mushroom poisoning

Гемолітико-уремічний синдром Hemolytic uremic syndrome

Отруєння грибами. Гостра ниркова недостатність Mushroom poisoning. Acute kidney failure

33 / 200
Хворий 37-ми років скаржиться на на- ростаючий приступ ядухи, кашель із незначним виділенням харкотиння, відчуття ''музики''в грудях. Об’єктивно: дихання утруднене, поверхневе, екскурсія грудної клітки обмежена, видих подовжений, на відстані чути сухі хрипи. ЧД- 14/хв. Пер-куторно над легенями - коробковий звук; аускультативно ослаблене везикулярне дихання, сухі хрипи. Тони серця приглушені, ритмічні, тахікардія. ЧСС- 100/хв. Який лікарський засіб необхідно ввести в першу чергу? A 37-year-old patient complains of an increasing attack of dyspnea, cough with slight expectoration, a feeling of 'music' in the chest. Objectively: breathing difficult, superficial, the excursion of the chest is limited, exhalation is prolonged, dry rales can be heard in the distance. BH- 14/min. Percussion over the lungs - a box sound; auscultation weakened vesicular breathing, dry rales. Heart sounds are muffled, rhythmic, tachycardia. Heart rate - 100/min. Which drug should be administered first?

Еуфілін Euphilin

Димедрол Diphenhydramine

Преднізолон Prednisone

Строфантин Strophantin

Адреналін Adrenaline

34 / 200
Жінка 35-ти років працювала на присадибній ділянці в спеку з непокритою головою. Через півтори години у неї з’явились слабкість, запаморочення, шум у вухах, порушення чіткості зору, почервоніння відкритих частин шкіри. Об’єктивно: ЧСС-1І0/хв., АТ- 90/60 мм рт.ст., ЧД- 28/хв., температура 38, 7oC. Які заходи необхідно провести на догоспітальному етапі? A 35-year-old woman worked in the yard in the heat with her head uncovered. After an hour and a half, she developed weakness, dizziness, tinnitus, impaired clarity vision, reddening of exposed parts of the skin. Objectively: heart rate - 1І0/min., blood pressure - 90/60 mm Hg, blood pressure - 28/min., temperature 38.7oC. What measures should be taken at the pre-hospital stage?

Обдати холодним душем Take a cold shower

Використати нейролептики Use neuroleptics

Дати випити теплий чай Give to drink warm tea

Дати антипіретики Give antipyretics

Помістити в тінь з холодним компресом на голові та напоїти холодною водою Place in the shade with a cold compress on the head and drink cold water

35 / 200
У хворого з політравмою на етапі транспортування до стаціонару раптово різко погіршився стан: непритомний, відсутність пульсу на центральних артеріях та самостійного дихання, розширились зіниці, шкірні покриви бліді, атонія, арефлексія, адинамія. Оцініть стан хворого: At the stage of transportation to the hospital, the patient with polytrauma suddenly deteriorated sharply: unconscious, lack of pulse on the central arteries and independent breathing, dilated pupils, pale skin, atony , areflexia, adynamia. Assess the patient's condition:

Біологічна смерть Biological death

Термінальна пауза Terminal Pause

Передагонія Peredagonia

Агонія Agony

Клінічна смерть Clinical death

36 / 200
Хворий 18-ти років звернувся за медичною допомогою до приймального відділення міської лікарні зі скаргами на нудоту, неодноразове блювання, пронос. З анамнезу відомо, що одну добу тому назад з’їв тістечко з білковим кремом. Об’єктивно: АТ- 90/60 мм рт.ст., Ps- 112/хв. Тургор шкіри знижений, діурез 600 мл на добу. Якими будуть лабораторні показники? An 18-year-old patient sought medical help at the reception department of a city hospital with complaints of nausea, repeated vomiting, diarrhea. It is known from the anamnesis that one day ago ate a cake with protein cream. Objectively: blood pressure - 90/60 mm Hg, Ps - 112/min. Skin turgor is reduced, diuresis 600 ml per day. What will the laboratory parameters be?

Гіперглікемія Hyperglycemia

Поліцитемія Polycythemia

Анемія Anemia

Агранулоцитоз Agranulocytosis

Тромбоцитопенія Thrombocytopenia

37 / 200
Хлопчик 11-ти років, що хворіє на цукровий діабет протягом 7-ми років, отри- мав 10 ОД простого інсуліну перед сніданком і 6 ОД перед обідом. В обід з’їв мало, через 30 хвилин після обіду знепритомнів, з’явилися судоми, блідість і виражена вологість шкіри, тризм щелеп, тони серця приглушені, тахікардія до 105/хв, артеріальна гіпотонія. З чого слід розпочати невідкладну допомогу? An 11-year-old boy with diabetes for 7 years received 10 units of simple insulin before breakfast and 6 units before dinner. I ate little lunch, 30 minutes after dinner I fainted, convulsions appeared, pallor and marked wetness of the skin, trismus of the jaws, muffled heart sounds, tachycardia up to 105/min, arterial hypotension. Where should emergency care be started?

Внутрішньовенне струминне введення 40% розчину глюкози Intravenous jet injection of 40% glucose solution

Підшкірне введення 0,1% розчину адреналіну Subcutaneous injection of 0.1% adrenaline solution

Внутрішньовенне краплинне введення 5% розчину глюкози Intravenous drip of 5% glucose solution

Внутрішньовенне введення глюкокорти-коїдів Intravenous administration of glucocorticoids

Внутрішньовенне введення 10% розчину хлориду натрію Intravenous administration of 10% sodium chloride solution

38 / 200
Вагітна в терміні 36-ти тижнів доставлена із скаргами на болі в низу живота, кров’янисті виділення із статевих шляхів. Об’єктивно: АТ- 150/100 мм рт.ст., набряки нижніх кінцівок. Серцебиття плоду 160/хв., приглушене. Вагінальне дослідження: шийка матки сформована, вічко замкнуте. Яка тактика лікаря? A 36-week pregnant woman was delivered with complaints of pain in the lower abdomen, bloody discharge from the genital tract. Objectively: blood pressure - 150/100 mm Hg, edema of the lower extremities. Fetal heartbeat 160/min., muffled. Vaginal examination: cervix is ​​formed, the eye is closed. What are the doctor's tactics?

Провести гемотрансфузію Perform blood transfusion

Почати пологозбудження Start labor induction

Лікувати гестоз Treat preeclampsia

Зробити терміново кесарський розтин Perform an urgent caesarean section

Ввести токолітики Enter tocolytics

39 / 200
Студент 21-го року доставлений бригадою швидкої допомоги до клініки у непритомному стані. Хворіє на цукровий діабет I типу. Впродовж 2-х днів скаржився на нудоту, блювання, діарею. Вранці ін’єкцію інсуліну не зробив, тому що не міг поснідати. Об’єктивно: хворий без свідомості, шкіра і слизові сухі, дихання глибоке, часте, шумне; АТ-100/60 мм рт.ст., Ps- 110/хв., рефлекси знижені. Які невідкладні заходи слід розпочати? A 21st-year student was brought to the clinic by an ambulance in an unconscious state. He suffers from type I diabetes. For 2 days, he complained of nausea, vomiting, diarrhea. He did not inject insulin in the morning because he could not have breakfast. Objectively: the patient is unconscious, the skin and mucous membranes are dry, breathing is deep, frequent, noisy; BP-100/60 mm Hg, Ps- 110 /min., reflexes are reduced. What emergency measures should be initiated?

Введення ізотонічного розчину в/в Introduction of isotonic IV solution

Введення пролонгованого інсуліну в/в зі швидкістю 5 ОД/год Introduction of long-acting insulin IV at a rate of 5 units/h

Ввести 2 мл кордіаміну в/м Enter 2 ml of cordiamine intravenously

Введення 5% глюкози 200 мл в/в Introduction of 5% glucose 200 ml IV

Введення простого інсуліну в/в зі швидкістю 5 ОД/год Administration of simple IV insulin at a rate of 5 units/h

40 / 200
Дівчинка народилася з масою 3400 г, з оцінкою за шкалою Апгар 8 балів. На 5-ту добу дитина була виписана додому. На 9-ту добу життя на шкірі дитини з’явилась везикулопустульозна висипка в місцях природніх складок. Загальний стан дитини не порушений. Загальний аналіз крові без змін. Який найбільш імовірний діагноз? The girl was born with a weight of 3400 g, with an Apgar score of 8 points. On the 5th day, the child was discharged home. On the 9th day of life on the skin the child developed a vesiculopustular rash in places of natural folds. The general condition of the child is not affected. The general blood test is unchanged. What is the most likely diagnosis?

Краснуха Krasnukha

Ексфолiативний дерматит Ріттера Ritter's exfoliative dermatitis

Епідермальна пухирчатка новонароджених, доброякісна форма Epidermal pemphigus of newborns, benign form

Вроджений сифіліс Congenital syphilis

Везикулопустульоз Vesiculopustulosis

41 / 200
Дитина 7-ми місяців раптом знепритомніла. З’явились тоніко-клонічні судоми, які тривали 2 хвилини. Спостерігалася ціано-тичність шкіри. Температура тіла нормальна. Після судоми дитина опритомніла, загальний стан задовільний. У крові: кальцій - 0,8 ммоль/л, фосфор - 1,3 ммоль/л. Яка причина судом? A 7-month-old child suddenly fainted. Tonic-clonic convulsions appeared that lasted 2 minutes. The skin was cyanotic. The body temperature was normal. After the seizure the child has passed out, the general condition is satisfactory. In the blood: calcium - 0.8 mmol/l, phosphorus - 1.3 mmol/l. What is the cause of seizures?

Травма мозку Brain injury

Менінгіт Meningitis

Гіпокальціємія, спазмофілія Hypocalcemia, spasmophilia

Об’ємний процес мозку Volume brain process

Некомпенсована гідроцефалія Uncompensated hydrocephalus

42 / 200
Пацієнт надійшов до приймального відділення лікарні в супроводі дружини. Впродовж 3-х років зловживає алкоголем. Об’єктивно: свідомість порушена, ціаноз шкірних покривів, температура 35, 5oC, різке зниження м’язового тонусу, ЧД- 12/хв., АТ-100/60 мм рт.ст., Ps- 60/хв. Різкий запах алкоголю з рота. Вміст алкоголю в крові -2,5o/oo. З чого слід розпочати лікування? The patient arrived at the reception department of the hospital accompanied by his wife. He has been abusing alcohol for 3 years. Objectively: consciousness is impaired, cyanosis of the skin, temperature 35.5oC , a sharp decrease in muscle tone, BH - 12/min., BP - 100/60 mm Hg, Ps - 60/min. Sharp smell of alcohol from the mouth. Blood alcohol content -2.5o/oo. With what should treatment be started?

Введення антибіотиків Introduction of antibiotics

Промивання шлунку Gastric lavage

Введення кардіотонічних засобів Introduction of cardiotonic drugs

Введення седативних засобів Introduction of sedatives

Введення проносних засобів Introduction of laxatives

43 / 200
Хворий 25-ти років прооперований у хірургічному відділенні з приводу апендициту. З анамнезу: через рік після травми, отриманої у 18-річному віці, почалися тоніко-клонічні напади із частотою 5-6 на рік. На 3-й день після операції розвинувся епілептичний статус: напади повторювались через 25 хвилин і між ними хворий не приходив до тями. Який препарат є засобом вибору у даному випадку? A 25-year-old patient was operated on in the surgical department for appendicitis. From the anamnesis: a year after the injury received at the age of 18, tonic-clonic seizures began with a frequency of 5-6 per year. On the 3rd day after the operation, status epilepticus developed: the attacks repeated after 25 minutes and between them the patient did not regain consciousness. What drug is the drug of choice in this case?

Сірчанокисла магнезія внутрішньом’язо-во Magnesium sulfate intramuscularly

Седуксен внутрішньовенно Seduxen intravenously

Аміназин внутрішньовенно Aminazine intravenously

Галоперидол внутрішньовенно Haloperidol intravenously

Бромід натрію внутрішньовенно Sodium bromide intravenously

44 / 200
У юнака 21-го року, хворого на хронічний вірусний гепатит С, з’явилися ейфорія, дезорієнтація, які змінилися сонливістю, сплутаною свідомістю. На шкірі - геморагічні висипання. Виникла носова кровотеча; з рота - солодкуватий запах. Яке ускладнення найбільш імовірне? A 21-year-old young man with chronic viral hepatitis C developed euphoria and disorientation, which were replaced by drowsiness and confusion. Hemorrhagic rashes on the skin . There was a nosebleed; a sweet smell from the mouth. What is the most likely complication?

Гостра ниркова недостатність Acute renal failure

Гостра печінкова недостатність Acute liver failure

Гіпоглікемічна кома Hypoglycemic coma

Гіпохлоремічна кома Hypochloremic coma

Геморагічний васкуліт Hemorrhagic vasculitis

45 / 200
Як розрахувати енергію першого розряду при проведенні дефібриляції дітям? How to calculate the energy of the first discharge when defibrillating children?

0,1 Дж/кг 0.1 J/kg

100 Дж 100 J

2 Дж/кг 2 J/kg

300 Дж 300 J

10 Дж/кг 10 J/kg

46 / 200
Хворий 25-ти років після падіння з дерева скаржиться на сильний біль у лівому стегні та лівій гомілці. Об’єктивно: гомілка та стегно деформовані, набряклі. На нозі імпровізована шина. Який препарат треба використати для зменшення больового синдрому? A 25-year-old patient complains of severe pain in the left thigh and left leg after falling from a tree. Objectively: the leg and thigh are deformed, swollen. On the leg improvised splint. What drug should be used to reduce the pain syndrome?

Анальгін Analgin

Диклофенак Diclofenac

Промедол Promedol

Баралгін Baralgin

ібупрофен ibuprofen

47 / 200
В поліклініку для проведення ФГДС натщесерце (їжу не приймав 12 годин) прийшов хворий 28-ми років. Протягом 15-ти років хворіє на цукровий діабет I типу. Раптово в реєстратурі знепритомнів. Виражені гіпорефлексія та потовиділення, тонус м’язів підвищений, періодами клоніко-тонічні судоми. АТ- 140/90 мм рт.ст. Ps-124/хв. Температура 36, 5oC. Який діагноз найбільш імовірний? A 28-year-old patient came to the polyclinic on an empty stomach (he had not eaten for 12 hours). He has been suffering from type I diabetes for 15 years. Suddenly fainted in the registry office. Pronounced hyporeflexia and sweating, increased muscle tone, periods of clonic-tonic convulsions. Blood pressure - 140/90 mmHg. Ps-124/min. Temperature 36.5oC. What is the most likely diagnosis?

Гіперглікемічна кома Hyperglycemic coma

Гіпоглікемічна кома Hypoglycemic coma

Печінкова кома Hepatic coma

Кетоацидотична кома Ketoacidotic coma

Алкогольна кома Alcoholic coma

48 / 200
Жінка 21-го року доставлена до пологового відділення швидкою допомогою з діагнозом: вагітність I, 40 тижнів, 5 нападів еклампсії вдома. Об’єктивно: стан важкий, кома II-III, дихання Чейн-Стокса. АТ-200/140 мм рт.ст. У сечі білок 5,8 г/л, ана-сарка. З чого необхідно почати реанімаційні заходи? A 21-year-old woman was brought to the maternity ward by ambulance with the diagnosis: pregnancy I, 40 weeks, 5 attacks of eclampsia at home. Objectively: the condition is severe, coma II-III, Cheyne-Stokes breathing. BP-200/140 mm Hg. Protein in the urine is 5.8 g/l, ana-sarka. Where should resuscitation measures be started?

Епідуральна анестезія Epidural anesthesia

Внутрішньовенно ввести 10 мг сибазону Enter 10 mg of sibazone intravenously

Термінове проведення ШВЛ на фоні барбітурового наркозу Urgent ventilation on the background of barbiturate anesthesia

Масковий наркоз фторотаном Fluorotan mask anesthesia

Внутрішньовенно ввести 25% розчин сірчанокислої магнезії зі швидкістю 2 г/год Enter 25% magnesium sulfate solution intravenously at a rate of 2 g/h

49 / 200
Після ДТП доставлений хворий з закритим переломом кісток тазу і правої стегнової кістки. Лікарем ШМД проведена транспортна іммобілізація і знеболювання. Через годину стан хворого різко погіршився: виражена блідість, холодний липкий піт, Ps слабкий, до 140 /хв., АТ- 60-70/0 мм рт.ст. Яка причина погіршення стану? After a road accident, a patient with a closed fracture of the pelvis and right femur was brought in. The doctor of ShMD carried out transport immobilization and anesthesia. An hour later, the patient's condition deteriorated sharply: marked pallor, cold sticky sweat, Ps is weak, up to 140 /min., BP - 60-70/0 mm Hg. What is the reason for the deterioration of the condition?

Кровотеча, що триває Ongoing bleeding

Тром6оєм6олія легеневої артерії Pulmonary artery thrombosis

Струс головного мозку Concussion

Гострий інфаркт міокарда Acute myocardial infarction

Травматичний шок Traumatic shock

50 / 200
Хвора 30-ти років, яка протягом 17-ти років страждає на цукровий діабет I типу з високими цифрами цукру в крові та частими коматозними станами, надійшла до хірургічного відділення у плановому порядку для проведення холецистектомії у зв’язку з жовчнокам’яною хворобою. Отримує інсулін тривалої дії (40 ОД/добу). Які заходи необхідні для попередження розвитку коматозного стану в післяопераційному періоді? A 30-year-old patient, who has been suffering from type I diabetes for 17 years with high blood sugar and frequent comatose states, came to the surgical department scheduled for cholecystectomy due to gallstone disease. Receives long-acting insulin (40 units/day). What measures are necessary to prevent the development of a comatose state in the postoperative period?

Збільшити дозу інсуліну тривалої дії Increase the dose of long-acting insulin

Відмінити інсулін Cancel insulin

До інсуліну додати манініл Add maninil to insulin

Зменшити дозу інсуліну тривалої дії Reduce dose of long-acting insulin

Призначити інсулін короткої дії, відмінивши інсулін тривалої дії Prescribe short-acting insulin, canceling long-acting insulin

51 / 200
У дитини з неправильною будовою зовнішніх геніталій раптово з’явились різка слабість артеріальна гіпотензія, ниткоподібний пульс, шкірні покриви мармурові, гіперпігментація сосків, зовнішніх статевих органів. Яка невідкладна допомога до уточнення діагнозу? A child with an abnormal structure of the external genitalia suddenly developed severe weakness, arterial hypotension, thread-like pulse, marbled skin, hyperpigmentation of the nipples, external genitalia. What emergency help to clarify the diagnosis?

Введення серцевих глікозидів в/в Introduction of cardiac glycosides IV

Введення гідрокортизону парентерально Introduction of parenteral hydrocortisone

Дати зволожений кисень Give humidified oxygen

Крапельне введення фізрозчину в/в Drip introduction of physiological solution IV

Введення адреналіну парентерально Adrenaline administration parenterally

52 / 200
Хворий 75-ти років скаржиться на неможливість здійснити сечовипуск впродовж 12-ти годин. При пальпації живота визначається збільшений сечовий міхур. Спроби провести катетеризацію сечового міхура гумовим та металевим катетерами безуспішні. Яка тактика лікаря? A 75-year-old patient complains of the inability to urinate for 12 hours. Palpation of the abdomen reveals an enlarged bladder. Attempts to catheterize the bladder with rubber and metal catheters unsuccessful. What are the doctor's tactics?

Помістити хворого в гарячу ванну Put the patient in a hot bath

Ввести хворому спазмолітики та знеболюючі препарати Give the patient antispasmodics and painkillers

Ввести спазмолітики та провести повторну катетеризацію сечового міхура гумовим катетером Introduce antispasmodics and re-catheterize the bladder with a rubber catheter

Провести електростимуляцію сечового міхура Perform electrical stimulation of the bladder

Провести надлобкову пункцію сечового міхура Perform a suprapubic puncture of the bladder

53 / 200
Лікар викликаний до дому до потерпілого 12-ти років. Скарги на біль у лівій половині грудної клітки, кровохаркання. Травма добової давнини. Об’єктивно: між 3 та 5 ребрами по передній аксилярній лінії ділянка припухлості і підшкірний кро- вовилив до 8 см в діаметрі. Яка подальша тактика? The doctor was called to the home of the 12-year-old victim. Complaints of pain in the left half of the chest, hemoptysis. Injury a day ago. Objectively: between 3 and 5 ribs along the front axillary line, the area of ​​swelling and subcutaneous hemorrhage up to 8 cm in diameter. What are the further tactics?

Госпіталізація до хірургічного відділення Hospitalization to the surgical department

Консультація травматолога в плановому порядку Consultation of a traumatologist as planned

Туге бинтування грудної клітки Tight bandaging of the chest

Лікування амбулаторне Outpatient treatment

Рентгенографія грудної клітки Chest X-ray

54 / 200
Хворий на хронічний гломерулонефрит після перенесеної ангіни відмітив зниження сечовиділення, появу нудоти, блювання, сонливість. З’явилися набряки обличчя. Об’єктивно: шкіра суха із слідами розчухувань. Креатинін крові - 820 мкм/л, клубо-чкова фільтрація - 10 мл/хв. Яким заходам слід надати перевагу? A patient with chronic glomerulonephritis after a sore throat noted a decrease in urination, the appearance of nausea, vomiting, drowsiness. Facial swelling appeared. Objectively: the skin is dry with traces of scratching Blood creatinine - 820 µm/l, glomerular filtration - 10 ml/min. What measures should be preferred?

Гемотрансфузія Hemotransfusion

Лімфоцитаферез Lymphocytapheresis

Гемосорбція Hemosorption

Гемодіаліз Hemodialysis

Плазмаферез Plasmapheresis

55 / 200
Постраждалий 28-ми років з пораненням колючим предметом передньої поверхні грудної клітки зліва. При диханні у ділянці рани - свистячий шум. Який діагноз і дії лікаря? A 28-year-old victim with a stabbing injury to the front surface of the chest on the left side. When breathing in the area of ​​the wound, there is a whistling noise. What is the doctor's diagnosis and actions?

Проникаюче поранення грудей. Обробка рани, суха пов’язка на рану, профілактика правцю, доставка до стаціонару Penetrating chest wound. Wound treatment, dry wound dressing, tetanus prevention, delivery to hospital

Проникаюче поранення грудей. Накладання стискаючої пов’язки, подача кисню, введення кордіаміну, доставка у терміновому порядку до стаціонару Penetrating chest wound. Compression bandage, oxygen administration, cordiamine administration, urgent delivery to hospital

Непроникаюче поранення грудної клітки зліва. Обробка рани, накладання пов’язки, доставка до стаціонару Non-penetrating wound of the chest on the left side. Treatment of the wound, applying a bandage, delivery to the hospital

Відкрите пошкодження грудей. Введення серцевих глікозидів, доставка до стаціонару Open chest injury. Administration of cardiac glycosides, delivery to hospital

Відкрите пошкодження грудей. Оцінка ступеня крововтрати, зупинка кровотечі, торакальна пункція, доставка до стаціонару Open injury to the chest. Assessment of the degree of blood loss, stop bleeding, thoracic puncture, delivery to the hospital

56 / 200
До лікарні поступила хвора на злоякісну пухлину непритомна 40-річна жінка, яка з метою самогубства ввела собі 200 мг морфіну. При огляді звуження зіниць, поверхневе рідке дихання, брадикардія. Який антидот слід призначити? An unconscious 40-year-old woman with a malignant tumor was admitted to the hospital, who injected herself with 200 mg of morphine with the intention of committing suicide. On examination, narrowing of the pupils, shallow shallow breathing, bradycardia . What antidote should be prescribed?

Налоксон Naloxone

Атропін Atropine

Дефероксамін Deferoxamine

Глюкагон Glucagon

Фізостигмін Physostigmine

57 / 200
У хворої 27-ми років раптово з’явились набряки шкіри і підшкірної клітковини в ділянці обличчя. Шкіра на обличчі спочатку була ціанотична, потім зблідла. Незначний свербіж. Об’єктивно: на шкірі тулуба поодинокі пухирчасті сверблячі висипання. Голос сиплий, сухий надсадний кашель. В ділянці живота незначна болючість. В анамнезі аналогічні стани різного ступеню інтенсивності, які самовільно щезали. Який найбільш імовірний діагноз? A 27-year-old patient suddenly developed swelling of the skin and subcutaneous tissue in the area of ​​the face. The skin on the face was cyanotic at first, then turned pale. Slight itching. About 'objectively: on the skin of the body there are isolated bubbling itchy rashes. The voice is hoarse, dry cough. There is slight pain in the abdomen. In the anamnesis, there are similar conditions of varying degrees of intensity, which disappeared spontaneously. What is the most likely diagnosis?

Синдром Мішера Mischer syndrome

Набряк Квінке Quincke edema

Бронхіальна астма Bronchial asthma

Анафілактичний шок Anaphylactic shock

Гострий живіт Acute stomach

58 / 200
Хвора 37-ми років, яка хворіє на ревматичну хворобу серця та комбіновану мі-тральну ваду серця, після ін’єкції серцевих глікозидів знепритомніла. Об’єктивно: свідомість відсутня, пульс на променевих і сонних артеріях не визначається, дихання поверхневе, шумне. Артеріальний тиск не визначається. Тони серця не вислуховуються. Який метод діагностики буде найін-формативнішим? A 37-year-old patient with rheumatic heart disease and combined mitral valve disease fainted after an injection of cardiac glycosides. Objectively: consciousness is absent, the pulse on the radial and carotid arteries is not determined, breathing is shallow, noisy. Blood pressure is not determined. Heart sounds are not heard. What method of diagnosis will be the most informative?

Коронароангіографія Coronary angiography

ЕКГ ECG

Комп’ютерна томографія органів грудної клітки Computer tomography of chest organs

Ro-графія органів грудної клітки Ro-graphy of chest organs

Ехо-КС Echo-KS

59 / 200
Дитина 5-ти років захворіла гостро -підвищилась температура тіла до 40oC, різкий біль при ковтанні, неможливість приймати навіть рідку їжу. Відмічається набряк мигдаликів, дужок, язичка. Набряк має дифузний характер без чітких меж і локальних випинань. Гіперемія слизових ротоглотки з ціанотичним відтінком. На мигдаликах тонкий, сіруватий неоднорідний наліт. Біль у ділянці шиї, збільшені і болючі регіонарні лімфатичні вузли. Який діагноз найбільш імовірний? A 5-year-old child became acutely ill - the body temperature rose to 40oC, sharp pain when swallowing, the inability to take even liquid food. Swelling of the tonsils, jaws, tongue is noted. The swelling has a diffuse character without clear boundaries and local protrusions. Hyperemia of the mucous membranes of the oropharynx with a cyanotic hue. On the tonsils, there is a thin, grayish inhomogeneous plaque. Pain in the neck, enlarged and painful regional lymph nodes. What is the most likely diagnosis?

Дифтерія мигдаликів, тяжка форма Diphtheria of the tonsils, severe form

Лакунарна ангіна Lacunar angina

Інфекційний мононуклеоз Infectious mononucleosis

Лімфогранулематоз Lymphogranulomatosis

Паратонзилярний абсцес Paratonsillar abscess

60 / 200
Хворий 43-х років, що хворіє на цукровий діабет протягом 20-ти років, потрапив до хірургічного відділення зі скаргами на біль в животі, нудоту, блювання. Об’єктивно: загальний стан тяжкий, свідомість спутана, тахікардія, гіпертензія, дихання Ку-смауля, гіперглікемія, ацетонурія. Хворому має бути проведене термінове оперативне втручання з приводу перфоративної виразки шлунка. Яку особливість слід врахувати анестезіологу? A 43-year-old patient, suffering from diabetes for 20 years, was admitted to the surgical department with complaints of abdominal pain, nausea, vomiting. About 'objectively: the general condition is severe, consciousness is confused, tachycardia, hypertension, Ku-Smaul breathing, hyperglycemia, acetonuria. The patient should undergo an urgent surgical intervention for a perforated stomach ulcer. What feature should the anesthesiologist take into account?

Дози препаратів для премедикації треба знижувати Doses of drugs for premedication should be reduced

II типу завжди необхідно периоперативне введення інсуліну Type II always requires perioperative insulin administration

Ендокринна відповідь на гіпоглікемію знижується при анестезії Endocrine response to hypoglycemia is reduced during anesthesia

в-блокатори не роблять анестезію більш небезпечною v-blockers do not make anesthesia more dangerous

Асистолія зустрічається не частіше, ніж у пацієнтів, що не страждають на діабет Asystole occurs no more often than in non-diabetic patients

61 / 200
Хвора 16-ти років захворіла 2 доби тому гостро з підвищення температури тіла до 37, 5oC. Скарги на погіршення апетиту та важкість у правому підребер’ї. Місяць тому була в контакті з хворими на вірусний гепатит A 16-year-old patient became acutely ill 2 days ago with an increase in body temperature to 37.5oC. Complaints of loss of appetite and heaviness in the right hypochondrium. A month ago was in contact with patients with viral hepatitis

Загальний білірубін та його фракції 6- Холестерин Total bilirubin and its fractions 6- Cholesterol

Кисла та лужна фосфатази Acid and alkaline phosphatases

Аланінамінотрансфераза, тимолова проба Alanine aminotransferase, thymol test

Об’єктивно: жовтяниці немає, печінка збільшена на 2 см. Сеча темна. Які біохімічні показники необхідно визначити для підтвердження безжовтяничної форми гепатиту? Objectively: there is no jaundice, the liver is enlarged by 2 cm. The urine is dark. What biochemical indicators should be determined to confirm the non-icteric form of hepatitis?

Електроліти K та Na K and Na electrolytes

62 / 200
У дитини 3-х років протягом останніх 2-х діб спостерігались млявість, зниження апетиту, нудота. Випорожнення водянисті 5-6 разів на добу, без патологічних домішок. Маса тіла знижена на 4%. Який метод регідратаційної терапії слід призначити? For the past 2 days, a 3-year-old child has experienced lethargy, decreased appetite, and nausea. Watery stools 5-6 times a day, without pathological impurities. Body weight is reduced by 4%. What method of rehydration therapy should be prescribed?

Внутрішньокістковий крапельний Intraosseous drip

Внутрішньовенний крапельний Intravenous drip

Внутрішньовенний струминний Intravenous jet

Оральний Oral

Підшкірний крапельний Subcutaneous drip

63 / 200
У процесі забезпечення анестезіологічного засобу дитині 6-ти років, оперованій з приводу розлитого перитоніту, проводиться штучна вентиляція легень у режимі помірної гіпервентиляції. На яких цифрах необхідно підтримувати рівень pCO2 у КЛС-грамі? In the process of providing an anesthetic agent to a 6-year-old child operated on for diffuse peritonitis, artificial lung ventilation is performed in the mode of moderate hyperventilation. At what numbers should the pCO2 level be maintained in the KLS-gram?

35-40 мм рт.ст. 35-40 mm Hg

30-35 мм рт.ст. 30-35 mm Hg

20-25 мм рт.ст. 20-25 mm Hg

50-55 мм рт.ст. 50-55 mm Hg

40-45 мм рт.ст. 40-45 mm Hg

64 / 200
Хлопчик 7-ми років, який впродовж року хворіє на цукровий діабет, у важкому стані госпіталізований до стаціонару. Важкість стану обумовлена розвитком гіпер-глікемічної коми. Яку дозу інсуліну необхідно ввести в першу годину надання невідкладної допомоги? A 7-year-old boy who has been suffering from diabetes for a year is hospitalized in a serious condition. The severity of the condition is due to the development of a hyperglycemic coma. What dose of insulin is it necessary to enter in the first hour of providing emergency care?

0,2 ОД/кг 0.2 U/kg

0,5 ОД/кг 0.5 units/kg

1,0 ОД/кг 1.0 U/kg

0,25 ОД/кг 0.25 units/kg

0,1 ОД/кг 0.1 U/kg

65 / 200
Хвора 30-ти років скаржиться на періодичне раптове підвищення артерiального тиску до 280/180 мм рт.ст., що супроводжується пітливістю, тремтінням всього тіла, почуттям страху смерті Після гіпертонічного кризу спостерігається поліурія, різка слабкість. У періодах між кризами артеріальний тиск 120/80 мм рт.ст. Які додаткові дослідження потрібно провести для встановлення діагнозу? A 30-year-old patient complains of a periodic sudden increase in blood pressure up to 280/180 mm Hg, which is accompanied by sweating, trembling of the whole body, a feeling of fear of death After a hypertensive crisis, there is polyuria, sharp weakness. In the periods between crises, the blood pressure is 120/80 mm Hg. What additional studies should be conducted to establish the diagnosis?

Дослідження вмісту трийодтироніну та тироксину Triiodothyronine and thyroxine content research

Дослідження вмісту ацетону в сечі Research of acetone content in urine

Дослідження вмісту катехоламінів крові та сечі (бажано під час, або одразу ж після кризи) Study of blood and urine catecholamine content (preferably during or immediately after a crisis)

Дослідження вмісту АКТГ та кортизолу Study of ACTH and cortisol content

Дослідження осмолярності крові Blood osmolarity study

66 / 200
У хворого 43-х років через 5 днів після купання в річці виник озноб, підвищилася температура до 38°C, з’явився біль у попереку та литкових м’язах; з 3-го дня хвороби сеча стала темною. На 4-й день температура 39,4°C, шкіра та склери іктеричні, обличчя гіперемоване, АТ-100/70 мм рт.ст., печінка +3 см, діурез - 300 мл. Які лабораторні дослідження слід призначити в цьому випадку? A 43-year-old patient developed chills 5 days after swimming in the river, the temperature rose to 38°C, pain in the lower back and calf muscles appeared' ulcers; from the 3rd day of the illness, the urine became dark. On the 4th day, the temperature was 39.4°C, the skin and sclera were icteric, the face was hyperemic, BP-100/70 mm Hg, liver +3 cm, diuresis - 300 ml. What laboratory tests should be prescribed in this case?

Посів крові на стерильність Blood culture for sterility

Реакція аглютинації-лізису лептоспір Leptospira agglutination-lysis reaction

Визначення анти-HAV IgM Definition of anti-HAV IgM

Кров на 'товсту краплю' Blood on a 'thick drop'

Реакція Відаля Vidal reaction

67 / 200
В селі протягом 3-х діб зареєстровано 6 випадків захворювання на холеру. Як показали епідеміологічні обстеження і лабораторні дослідження, джерелом ураження холерою була питна вода. При проведенні профілактичних заходів які контингенти населення підлягають першочерговому лабораторному обстеженню? 6 cases of cholera were registered in the village over the course of 3 days. As shown by epidemiological examinations and laboratory studies, the source of cholera was drinking water. When carrying out preventive measures which contingents of the population are subject to priority laboratory examination?

З дисфункцією шлунково-кишкового тракту With gastrointestinal tract dysfunction

Ті, хто вживав сиру воду Those who drank raw water

Ті, хто спілкувався з хворими Those who communicated with the sick

Доросле населення Adult population

Дитяче населення Child population

68 / 200
Хлопчик 10-ти років, що страждає на гемофілію, доставлений до лікарні зі скаргами на набряк правого колінного суглоба, який виник 2 години тому після невеликої травми. Який обсяг медичної допомоги слід призначити хворому? A 10-year-old boy with hemophilia is brought to the hospital with complaints of swelling of the right knee joint, which occurred 2 hours ago after a minor injury. What volume should medical aid be prescribed to the patient?

Введення амінокапронової кислоти Introduction of aminocaproic acid

Повторні трансфузії антигемофільного фактору VIII 10 ОД/кг 1 раз в 12 годин, ліжковий режим, при різкому больовому синдромі - пункція суглоба Repeated transfusions of antihemophilic factor VIII 10 units/kg once every 12 hours, bed rest, with acute pain - joint puncture

Переливання нативної плазми Transfusion of native plasma

Холод на уражений суглоб, його іммобілізація Cold on the affected joint, its immobilization

Пряме переливання крові від донора Direct blood transfusion from a donor

69 / 200
У хворого 36-ти років настала клінічна смерть. З чого слід розпочати реанімаційні заходи? A 36-year-old patient has clinically died. What should resuscitation measures be started?

Вимірювання артеріального тиску Blood pressure measurement

Електрична дефібриляція Electrical defibrillation

Зовнішній масаж серця та ШВЛ різними способами External heart massage and ventilation in various ways

Адреналін внутрішньосерцево Adrenaline intracardiac

Електрокардіографія Electrocardiography

70 / 200
До прибуття бригади ШМД потерпілого витягли з зашморгу. Об’єктивно: непритомний, на шиї - бліда странгуляційна борозна. Виражене рухове збудження. Обличчя набрякле, синюшне. ЧД- 30/хв. В легенях - вологі хрипи середнього калібру. ЧСС- 140/хв. Екстрасистолія. АТ- 90/60 мм рт.ст. Який найбільш імовірний діагноз? Prior to the arrival of the fire brigade, the victim was pulled out of the shack. Objectively: unconscious, a pale strangulation furrow on the neck. Pronounced motor excitement. The face is swollen, bluish. ChD- 30/min. In the lungs - wet rales of medium caliber. Heart rate - 140/min. Extrasystole. BP - 90/60 mm Hg. What is the most likely diagnosis?

Странгуляційна асфіксія, ускладнена інсультом Strangulation asphyxia complicated by stroke

Странгуляційна асфіксія, ускладнена набряком легень Strangulation asphyxia complicated by pulmonary edema

Странгуляційна асфіксія Strangulation asphyxiation

Странгуляційна асфіксія, ускладнена ТЕЛА Strangulation asphyxia complicated by TELA

Странгуляційна асфіксія, ускладнена дисциркуляторною енцефалопатією Strangulation asphyxia complicated by dyscirculatory encephalopathy

71 / 200
Хворого з діагнозом дифтерія гортані госпіталізовано. Об’єктивно: ціаноз, інспі-раторна задишка, пульс аритмічний, ниткоподібний, шкіра вкрита потом. Стан хворого вкрай важкий; збуджений. Які дії лікаря є першочерговими? A patient with a diagnosis of diphtheria of the larynx was hospitalized. Objectively: cyanosis, inspiratory shortness of breath, pulse arrhythmic, threadlike, skin covered with sweat. The patient's condition is extremely difficult; excited . What actions of the doctor are of primary importance?

Внутрішньовенне введення 30-40 тис. МО ПДС Intravenous administration of 30-40 thousand IU of PDS

Введення антибіотиків широкого спектра дії Introduction of broad-spectrum antibiotics

Внутрішньовенне введення 20-30 тис. МО ПДС Intravenous administration of 20-30 thousand IU of PDS

Введення седативних препаратів та глю-кокортикостероїдів Introduction of sedatives and glucocorticosteroids

Проведення інтубації (трахеостомії) Intubation (tracheostomy)

72 / 200
Дитина 5-ти років надійшла до стаціонару вночі непритомна. Спостерігається гіпотонія, гіпорефлексія, шкіра блідо-сіра, суха, тургор тканин і очних яблук знижений. З анамнезу: ввечері дитина їла сметану з варениками. Вночі вона прокинулася від сильних болів у животі, відмічалося багаторазове блювання, запах ацетону з рота, психомоторне збудження, яке згодом змінилося вираженою слабкістю. Який вид коми розвинувся у дитини? A 5-year-old child came to the hospital unconscious at night. Hypotonia, hyporeflexia, pale gray, dry skin, reduced turgor of tissues and eyeballs are observed. From the history: in the evening, the child ate sour cream with dumplings. At night, she woke up from severe abdominal pain, repeated vomiting, the smell of acetone from her mouth, psychomotor excitement, which later changed to pronounced weakness. What kind of coma developed in the child?

Уремічна Uremic

Гіпохлоремічна Hypochloremic

Ацетонемічна Acetonemic

Наднирникова Adrenal

Гіпоглікемічна Hypoglycemic

73 / 200
Хворий 65-ти років захворів гостро: підвищилась температура до 38oC, з’явився біль в низу живота, часті рідкі випорожнення з домішками слизу, прожилками крові та гною у невеликій кількості. Перед актом дефекації тенезми. Який найбільш імовірний діагноз? A 65-year-old patient became acutely ill: the temperature rose to 38oC, there was pain in the lower abdomen, frequent loose stools with impurities of mucus, veins of blood and pus in a small amount. Before the act of tenesmus defecation. What is the most likely diagnosis?

Амебіаз Amebiasis

Сальмонельоз Salmonellosis

Загострення геморою Exacerbation of hemorrhoids

Черевний тиф Typhoid

Дизентерія Dysentery

74 / 200
У хворого протягом декількох годин спостерігаються часті судомні напади, впродовж яких він знаходиться в одному положенні, погляд спрямований в одну точку, свідомість весь час порушена. Яка тактика? The patient has frequent convulsive attacks for several hours, during which he is in one position, his gaze is directed at one point, consciousness is disturbed all the time. What are the tactics?

Госпіталізація в реанімаційне відділення Hospitalization in intensive care unit

Призначення консультації епілептолога Epileptologist consultation appointment

Госпіталізація в психіатричне відділення Hospitalization in a psychiatric ward

Призначення амбулаторного лікування Appointment of outpatient treatment

Госпіталізація в неврологічне відділення Hospitalization in the neurological department

75 / 200
Наркоман, який вживає героїн, на другий день після апендектомії почав скаржитися на сильні болі в суглобах, блювання, пронос, сухість в роті, безсоння, 'тремтіння всього тіла'. Об’єктивно: агресивний, злий, напружений. Настрій пригнічений -заявляє, що 'покінчить з собою'. Визначить психопатологічний синдром: A heroin addict, on the second day after appendectomy, began to complain of severe pain in the joints, vomiting, diarrhea, dry mouth, insomnia, 'tremors of the whole body '. Objectively: aggressive, angry, tense. The mood is depressed - he declares that he will 'kill himself'. Defines a psychopathological syndrome:

Істеричний Hysterical

Психопатичний Psychopathic

Депресивний Depressed

Абстинентний Abstinent

Дисфоричний Dysphoric

76 / 200
Новонароджений народився у стані асфіксії III ступеня. Дихання та серцебиття відсутнє. Якими повинні бути першочергові дії лікаря? A newborn was born in a state of III degree asphyxia. There is no breathing and heartbeat. What should be the primary actions of the doctor?

- -

Комбінований масаж серця Combined heart massage

Закритий масаж серця Closed heart massage

Ввести внутрішньосерцево 0,1% р-н адреналіну Inject intracardially 0.1% adrenaline

Не проводити масаж серця Do not massage the heart

77 / 200
У жінки 24-х років народилася доношена дитина з масою 4200 гр., оцінка за шкалою Апгар 4 бали. Самостійне дихання відсутнє, ЧСС- 80/хв., шкіра ціанотична. Відразу розпочата первинна реанімація. Яку концентрацію кисню слід використовувати при штучній вентиляції? A 24-year-old woman gave birth to a full-term baby weighing 4,200 grams, with an Apgar score of 4. No spontaneous breathing, heart rate 80/min. the skin is cyanotic. Primary resuscitation is started immediately. What concentration of oxygen should be used for artificial ventilation?

21% 21%

60-40% 60-40%

90-100% 90-100%

50% 50%

80-70% 80-70%

78 / 200
Хвора 45-ти років скаржиться на появу протягом двох тижнів випорожнень кров’ю більш за 12 разів на добу, болю в животі, суглобах; болючі висипки у порожнині рота та нижніх кінцівках, набряк колінних та гомілковостопних суглобів, слабкість, підвищення температури тіла понад 39oC, втрату ваги тіла. Який метод обстеження є найбільш інформативним? A 45-year-old patient complains of the appearance of bloody stools more than 12 times a day, pain in the abdomen, joints, painful rashes in the oral cavity for two weeks and lower limbs, swelling of knee and ankle joints, weakness, increase in body temperature above 39oC, loss of body weight. Which examination method is the most informative?

Ендоскопічне дослідження Endoscopy examination

Рентгенологічне дослідження кишечнику X-ray examination of intestines

Біопсія кишечнику та шкіри Intestinal and skin biopsy

Рентгенографія суглобів X-ray of joints

Загальний аналіз крові General blood test

79 / 200
62- літній чоловік з інфарктом міокарда в анамнезі прокинувся з вираженою правобічною геміплегією. Очі тонічно повернені вліво й не реагують на загрозливі дії з боку правого поля зору. Реакція на біль у лівій половині тіла збережена. Мова незрозуміла; наказів не виконує. Відтворити прості фрази не може. Який найбільш імовірний діагноз? A 62-year-old man with a history of myocardial infarction woke up with pronounced right-sided hemiplegia. The eyes are tonically turned to the left and do not react to threatening actions from the right visual field. Reaction to pain in the left half of the body is preserved. Speech is unclear; does not follow orders. Can not reproduce simple phrases. What is the most likely diagnosis?

Геморагічний інсульт Hemorrhagic stroke

- -

Менінгіт Meningitis

Атеросклероз Atherosclerosis

Ішемічний інсульт Ischemic stroke

80 / 200
Жінка 25-ти років звернулась до лікаря зі скаргами на незначні кров’янисті виділення з піхви протягом доби, болі в низу живота. Остання нормальна менструація 2 місяці тому. Об’єктивно загальний стан не порушений. Живіт при пальпації м’який, болючий в нижніх відділах. Виділення кров’янисті незначні. Яке додаткове обстеження є доцільним? A 25-year-old woman consulted a doctor with complaints of minor vaginal bleeding during the day, pain in the lower abdomen. The last normal menstruation was 2 months ago. Objectively, the general condition is not disturbed. The abdomen is soft on palpation, painful in the lower parts. Blood discharge is insignificant. What additional examination is appropriate?

Ультразвукове обстеження Ultrasound examination

Лапароскопічне обстеження Laparoscopic examination

Імунологічний тест на вагітність Immunological pregnancy test

Пункція заднього склепіння піхви Puncture of the posterior vault of the vagina

Рентгенологічне обстеження X-ray examination

81 / 200
Хворий скаржиться на біль в епігастрії, виражену слабкість, нудоту, блювання 'кавовою гущею', 'мерехтіння мушок'перед очима, випорожнення темного кольору. Об’єктивно: загальний стан важкий, температура тіла 37,4oC, АТ- 90/60 мм рт.ст., ЧСС- 115/хв. Тони серця ослаблені, систолічний шум. Живіт чутливий під час пальпації в епігастрії. Яка тактика лікаря? The patient complains of pain in the epigastrium, severe weakness, nausea, vomiting of 'coffee grounds', 'flickering of flies' in front of the eyes, dark-colored stools. Objectively: the general condition is serious, body temperature is 37.4oC, blood pressure - 90/60 mm Hg, heart rate - 115/min. Heart sounds are weakened, systolic murmur. Abdomen is sensitive during palpation in the epigastrium. What are the doctor's tactics?

Направити в поліклініку для дообсте-ження Send to the polyclinic for further examination

Направити хворого в інфекційне відділення Send the patient to the infectious department

Направити хворого в хірургічне відділення Send the patient to the surgical department

Направити хворого в терапевтичне відділення Send the patient to the therapeutic department

Надати невідкладну допомогу і рекомендувати амбулаторне лікування Provide emergency care and recommend outpatient treatment

82 / 200
У хворого 46-ти років під час їжі, раптово, серед повного здоров’я, виник напад ядухи, що супроводжується сильним кашлем, осиплістю голосу, афонією, болем у горлі. Об’єктивно: хворий неспокійний, шкірні покриви бліді, вологі, акроціаноз. Ps- 108/хв., ритмічний. Який попередній діагноз? A 46-year-old patient, while eating, suddenly, in full health, had an attack of dyspepsia, accompanied by a strong cough, hoarseness of voice, aphonia, pain in the throat. Objectively: the patient is restless, the skin is pale, moist, acrocyanosis. Ps- 108/min., rhythmic. What is the previous diagnosis?

Астматичний статус Asthmatic status

Напад бронхіальної астми Bronchial asthma attack

Бронхоспазм Bronchospasm

Стороннє тіло верхніх дихальних шляхів Upper respiratory tract foreign body

Ларингоспазм Laryngospasm

83 / 200
Хворий 68-ми років захворів гостро: різко, 'як удар', з’явився головний біль, підвищення температури. Об’єктивно: різко позитивні менінгеальні знаки. Які діагностичні заходи необхідно провести в першу чергу для встановлення діагнозу? A 68-year-old patient became acutely ill: suddenly, 'like a blow', a headache appeared, a rise in temperature. Objectively: sharply positive meningeal signs. What diagnostic measures must be carried out in the first place to establish a diagnosis?

Ехоенцефалографія Echoencephalography

Дослідження згортувальної системи крові Study of blood coagulation system

Загальний аналіз крові General blood test

Люмбальна пункція Lumbar puncture

Комп’ютерна томографія Computed tomography

84 / 200
Стан хворого на гострий гепатит В важкий. Збільшилась інтенсивність жовтяниці. Безсоння вночі, сонливість удень, потім з’явилося збудження. Об’єктивно: Ps-100/хв, АТ90 /60 мм рт.ст., печінка не пальпується, сухожилкові рефлекси підвищені. Діурез знижений. Який симптом є найбільш значущим при оцінюванні важкості стану хворого? The condition of a patient with acute hepatitis B is severe. The intensity of jaundice has increased. Insomnia at night, drowsiness during the day, then excitement appeared. Objectively: Ps-100/min , blood pressure 90 /60 mm Hg, the liver is not palpable, tendon reflexes are increased. Diuresis is reduced. Which symptom is the most significant when assessing the severity of the patient's condition?

Гіперрефлексія Hyperreflection

Зниження діурезу Diuresis decrease

Зменшення розмірів печінки Reduction in liver size

Інтенсивність жовтяниці Jaundice intensity

Гіпотонія Hypotonia

85 / 200
Хворий надійшов до стаціонару із клінікою печінкової недостатності. При огляді виявлено незначні неврологічні зрушення. Втрата ваги незначна. Наявний асцит легко піддається лікуванню. У крові: білірубін в сироватці крові - 3,0 мг/%, рівень альбуміну - 3,2 мг/%, протромбіновий індекс -75%. До якої групи печінково-клітинної недостатності можна віднести дані зрушення згідно з класифікацією печінкової дисфункції за Чайлд-П’ю? The patient was admitted to the hospital with a liver failure clinic. Examination revealed minor neurological changes. Weight loss is minor. The existing ascites is easily treatable. In the blood: bilirubin in the blood serum - 3.0 mg/%, albumin level - 3.2 mg/%, prothrombin index -75%. To which group of hepatocellular insufficiency can these shifts be attributed according to the classification of hepatic dysfunction according to Child-Pugh?

1 1

2 2

1 б 1 b

3 3

1 а 1 a

86 / 200
Хворий 35-ти років, мисливець, захворів гостро: з’явились різкі болі в правій пахвинній ділянці, які призвели до вимушеного положення руки, температура 39, 5°C. На наступний день викликав дільничного лікаря, який поставив діагноз лімфаденіт і машиною швидкої допомоги направив хворого до чергового хірургічного стаціонару. Хірург при огляді виявив дерев’янистої щільності конгломерат лімфатичних вузлів та багряно-синюшний колір шкіри над ним. Який найбільш імовірний діагноз? A 35-year-old patient, a hunter, became acutely ill: sharp pains appeared in the right groin area, which led to a forced hand position, temperature 39.5° C. The next day, he called the district doctor, who made a diagnosis of lymphadenitis and sent the patient to the next surgical hospital by ambulance. During the examination, the surgeon found a conglomerate of lymph nodes with a woody density and a purplish-bluish color of the skin above it. What is the most likely diagnosis?

Сепсис, гнійний лімфаденіт Sepsis, purulent lymphadenitis

Туляремія, бубонна форма Tularemia, bubonic form

Чума, бубонна форма Plague, bubonic form

Гнійний лімфаденіт Suppurative lymphadenitis

Тромбоз стегнової артерії Femoral artery thrombosis

87 / 200
До приймального відділення лікарні доставлено хворого після автомобільної травми. Об’єктивно: шкіра бліда, АТ-100/60 мм рт.ст., Ps- 140/хв. Який найбільш імовірний діагноз? A patient was brought to the hospital's admissions department after a car injury. Objectively: pale skin, BP-100/60 mm Hg, Ps- 140/min . What is the most likely diagnosis?

Септичний шок Septic shock

Множинні переломи ребер Multiple rib fractures

Травматичний шок Traumatic shock

Кардіогенний шок Cardiogenic shock

Анафілактичний шок Anaphylactic shock

88 / 200
В момент пологів у амніотичній рідині виявлено густий меконій. Відсмоктаний вміст з рота, глотки, носа відразу після народження голівки (до народження плеч). Анестезіолог відсмоктав вміст гіпофарин-ксу. Яка наступна дія лікаря? At the time of delivery, thick meconium was found in the amniotic fluid. The contents were sucked from the mouth, pharynx, and nose immediately after the birth of the head (before the birth of the shoulders). The anesthesiologist sucked the contents of the hypopharynx ksu. What is the doctor's next action?

Проведення тактильної стимуляції дихання Carrying out tactile stimulation of breathing

Оцінка кольору шкіри Skin Color Score

Інтубація трахеї і відсмоктування вмісту через ендотрахеальну трубку Intubation of the trachea and suction of contents through an endotracheal tube

Визначення частоти серцевих скорочень Determining heart rate

Штучна вентиляція легень з використанням мішка і маски Artificial lung ventilation using bag and mask

89 / 200
Хворий 56-ти років звернувся до клініки зі скаргами на біль в ділянці промежини та анального отвору, підвищення температури до 39°C, головний біль, загальне нездужання. Об’єктивно: на відстані 3 см від анального отвору болючий інфільтрат, який розповсюджується в анальний канал до перехідної складки слизової, з флюкту-ацією в центрі. Який найбільш імовірний діагноз? A 56-year-old patient came to the clinic with complaints of pain in the perineum and anus, temperature rise to 39°C, headache, general malaise. About objectively: at a distance of 3 cm from the anus, a painful infiltrate that spreads into the anal canal to the transitional fold of the mucous membrane, with fluctuations in the center. What is the most likely diagnosis?

Гострий підслизовий парапроктит Acute submucous paraproctitis

Гострий промежинний підшкірний парапроктит Acute perineal subcutaneous paraproctitis

Нєспєцифічний виразковий коліт Nonspecific ulcerative colitis

Гострий геморой Acute hemorrhoids

Хронічний парапроктит Chronic paraproctitis

90 / 200
У підлітка 15-ти років з виразковою хворобою шлунка 2 доби блювання з кров’ю, ''дьогтеподібне''випорожнення. Огляд: у свідомості, млявий, блідий. Симптом 'білої плями'2 секунди. Кінцівки теплі до ліктьових та колінних суглобів. Ps-104/хв., АТ- 90/50 мм рт.ст. Діурез знижений. Якою повинна бути базисна терапія до консультації хірурга? A 15-year-old boy with peptic ulcer has vomited blood for 2 days, tarry stools. Examination: conscious, lethargic, pale . Symptom of a 'white spot' for 2 seconds. The extremities are warm to the elbow and knee joints. Ps-104/min., BP- 90/50 mm Hg. Diuresis is reduced. What should be the basic therapy before consulting the surgeon?

Введення серцевих глікозидів Introduction of cardiac glycosides

Преднізолон внутрішньом’язово, амінокапронова кислота per os Prednisolone intramuscularly, aminocaproic acid per os

Адреналін внутрішньом’язово, амінокапронова кислота per os Adrenaline intramuscularly, aminocaproic acid per os

Внутрішньовенне краплинне введення реополіглюкіну Intravenous drip administration of rheopolyglucin

Вікасол внутрішньом’язово, амінокапронова кислота per os Vikasol intramuscularly, aminocaproic acid per os

91 / 200
У хворого 17-ти років через три тижні після ангіни з’явився гострий біль у колінних суглобах, підвищення температури тіла до 38oC. При огляді хворого: дефігурація та припухлість колінних суглобів з гіперемією шкіри над ними; найменший рух викликає гострий біль в уражених суглобах. Яка невідкладна допомога в амбулаторних умовах? Three weeks after angina, a 17-year-old patient developed acute pain in the knee joints, an increase in body temperature to 38oC. Upon examination of the patient: disfigurement and swelling knee joints with hyperemia of the skin over them; the slightest movement causes sharp pain in the affected joints. What is the emergency care in an outpatient setting?

Введення диклофенаку 3 мл в/м Introduction of diclofenac 3 ml IV

Введення реланіуму 2 мл в/м Introduction of relanium 2 ml IV

Введення но-шпи 1 мл в/м Introduction of no-spi 1 ml IV

Введення димедролу 1 мл в/м Introduction of Diphenhydramine 1 ml IV

Прийом парацетамолу 1 таб внутрішньо Taking paracetamol 1 tab internally

92 / 200
Хвора 76-ти років доставлена з гіпер-тензивним кризом. Об’єктивно: шкірні покриві бліді, набряклість обличчя, повік, гомілок. АТ - 180/120 мм рт.ст., ЧСС- б8/хв. На комбінації яких препаратів слід зупинити свій вибір? A 76-year-old patient was brought in with a hypertensive crisis. Objectively: pale skin, swelling of the face, eyelids, lower leg. Blood pressure - 180/120 mm Hg, heart rate - b8/min. What combination of drugs should you choose?

Венорутон + фуросемід Venoruton + furosemide

Строфантин + фуросемід Strophantin + furosemide

Ізосорбіду динітрат + фуросемід Isosorbide dinitrate + furosemide

Ніфедипін + фуросемід Nifedipine + furosemide

Нітрогліцерин + фуросемід Nitroglycerin + furosemide

93 / 200
У дитини 1-го року протягом трьох днів відзначаються часті рідкі випорожнення, повторне блювання. Діагностована кишкова інфекція, токсико-ексикоз ІІ ст. Яке лабораторне дослідження необхідно першочергово виконати для проведення адекватної інфузійної терапії? A child of the 1st year has had frequent liquid stools for three days, repeated vomiting. Diagnosed intestinal infection, toxic-exicosis of the 2nd degree. What laboratory test should be performed as a priority for adequate infusion therapy?

Визначення рівня цукру крові Blood sugar determination

Визначення білкового спектру крові Determination of protein spectrum of blood

Визначення імунного статусу Definition of immune status

Визначення електролітів крові Determination of blood electrolytes

Визначення активності ферментів печінки Determining the activity of liver enzymes

94 / 200
У хворого 63-х років вночі раптово почався сильний біль у поперековій ділянці, бічних відділах живота, який іррадіював у статевий орган, внутрішню поверхню правого стегна; нудота, часті поклики на сечовипускання, сеча червона зі згустками крові. Напад тривав більше 3-х годин. При обстеженні хворого визначена болючість та напруження м’язів правої половини живота, позитивний симптом Пастернацького справа. Який найбільш імовірний діагноз? A 63-year-old patient suddenly developed severe pain at night in the lumbar region, lateral parts of the abdomen, which radiated to the genital organ, the inner surface of the right thigh; nausea, frequent urges to urinate, urine is red with blood clots. The attack lasted more than 3 hours. During the examination of the patient, pain and tension of the muscles of the right half of the abdomen were determined, a positive Pasternatsky symptom on the right. What is the most likely diagnosis?

Гострий апендицит Acute appendicitis

Сечокам’яна хвороба (ниркова колька) Urolithiasis (renal colic)

Жовчнокам’яна хвороба (печінкова колька) Cholelithiasis (hepatic colic)

Кишкова непрохідність Intestinal obstruction

Гострий правобічний пієлонефрит Acute right-sided pyelonephritis

95 / 200
Чоловік 30-ти років доставлений в приймальне відділення після утоплення у прісній воді, яке відбулося близько 40 хвилин тому і супроводжувалося успішною реанімацією. Чоловік у свідомості, при аускультації у легенях вислуховуються хрипи над всіма легеневими полями. Основний напрямок подальшої терапії: A 30-year-old man was brought to the reception department after drowning in fresh water, which occurred about 40 minutes ago and was accompanied by successful resuscitation. The man is conscious, on auscultation in in the lungs, rales are heard over all lung fields. The main direction of further therapy:

Введення глюкокортикоїдів Introduction of glucocorticoids

Введення інгібіторів протеолізу Introduction of proteolysis inhibitors

Відновлення ОЦК Recovery of BCC

Дегідратація Dehydration

Киснетерапія Oxygen therapy

96 / 200
У новонародженої дитини від 2-ї вагітності через 12 годин після народження виявлено іктеричне забарвлення шкіри та склер. Об’єктивно: загальний стан швидко погіршується, печінка виступає з-під реберної дуги на 3 см, селезінка - на 2 см. В динаміці значне зростання білірубіну впродовж 2-х діб з 90 мкмоль/л до 150 мкмоль/л, переважно за рахунок непрямого. Яка тактика лікування? In a newborn child from the 2nd pregnancy, icteric coloration of the skin and sclera was detected 12 hours after birth. Objectively: the general condition is rapidly deteriorating, the liver protrudes from under the costal arch by 3 cm, the spleen by 2 cm. In the dynamics, a significant increase in bilirubin over the course of 2 days from 90 μmol/l to 150 μmol/l, mainly due to indirect. What are the tactics of treatment?

Призначення глюкокортикоїдів Prescription of glucocorticoids

Призначення гепатопротекторів Prescription of hepatoprotectors

Дезінтоксикаційна терапія Detoxification therapy

Противірусні препарати Antiviral drugs

Замінне переливання крові Replacement blood transfusion

97 / 200
Під час обстеження в 34 тижні вагітності у жінки виявляють стійку фетальну брадикардію і проводять кесарів розтин. У новонародженого відсутні дихання і м’язовий тонус. Вентиляція мішком і маскою не забезпечує адекватні рухи грудної клітки, і після наступних 30 секунд вентиляції ЧСС не збільшується. Який наступний етап реанімації? During the examination at the 34th week of pregnancy, persistent fetal bradycardia is detected in a woman and a cesarean section is performed. The newborn has no breathing and muscle tone. Bag and mask ventilation is not provides adequate chest movements, and after the next 30 seconds of ventilation, the heart rate does not increase. What is the next stage of resuscitation?

Припинити надання допомоги Stop providing assistance

Введення адреналіну Adrenaline injection

Введення натрію бікарбонату Introduction of sodium bicarbonate

Інтубація трахеї немовляти Tracheal intubation of an infant

Введення атропіну Atropine administration

98 / 200
Хворий, який знаходиться у відділєнні реанімації, раптово зблід і знепритомнів. Пульс на магістральних артеріях відсутній. На екрані кардiомонiтора з’явилась ізолінія. Який найбільш імовірний діагноз? A patient in the intensive care unit suddenly turned pale and fainted. There was no pulse on the main arteries. An isoline appeared on the heart monitor screen. What is the most likely diagnosis?

Електромеханічна дисоціація Electromechanical dissociation

Тріпотіння передсердь Atrial flutter

Асистолія Asystole

Фібриляція шлуночків Ventricular fibrillation

Миготлива аритмія Atrial fibrillation

99 / 200
У вагітної в 34 тижні виявляють стійку фетальну брадикардію і проводять операцію кесарева розтину. У новонародженого відсутні дихання і м’язовий тонус. Починається вентиляція легень киснем під позитивним тиском. Після 30 секунд вжитих заходів виявляється: ЧСС- 20-30/хв., рухи відсутні, залишається ціаноз. Який наступний етап реанімаційної допомоги? Persistent fetal bradycardia is detected in a pregnant woman at 34 weeks and a cesarean section is performed. The newborn has no breathing and muscle tone. Ventilation of the lungs with oxygen under positive pressure begins. After 30 seconds of the measures taken, the following is found: heart rate - 20-30/min., no movements, cyanosis remains. What is the next stage of resuscitation?

Введення натрію бікарбонату Introduction of sodium bicarbonate

Припинити надання допомоги Stop providing assistance

Введення адреналіну Injection of adrenaline

Інтубація трахеї немовляти Intubation of a baby's trachea

Непрямий масаж серця з вентиляцією легень під позитивним тиском Indirect heart massage with positive pressure ventilation

100 / 200
У дитини 1,5 місяців раптово погіршився стан: з’явились блювання, адинамія, розлади стулу. Об’єктивно: стан тяжкий, шкіра бліда із мармуровим малюнком, різка млявість, ознаки токсикоекси-козу, неправильна інтерсексуальна будова зовнішніх геніталій, та їх гіперпігментація. Пульс до 160/хв., слабкий. АТ- 40/20 мм рт.ст., температура тіла - 36, 5 oC. Призначення яких препаратів найбільш обґрунтовано? A 1.5-month-old child's condition suddenly worsened: vomiting, adynamia, stool disorders appeared. Objectively: the condition is severe, the skin is pale with a marble pattern, sharp lethargy, signs of toxicoexi-goat, irregular intersexual structure of the external genitalia, and their hyperpigmentation. Pulse up to 160/min., weak. Blood pressure - 40/20 mm Hg, body temperature - 36.5 oC. Prescribing which drugs are the most justified?

Антибіотики і детоксикаційні засоби Antibiotics and detoxification agents

Ілюкозо-сольові розчини Ilucose-saline solutions

Ілюко- і мінералокортикоїди Iluco- and mineralocorticoids

5% глюкоза і реополіглюкін 5% glucose and rheopolyglucin

Фізрозчин і реосорбілакт Physical solution and reosorbilact

101 / 200
У хворого з множинними переломами обох нижніх кінцівок на 2-гу добу розвинулись задишка, ціаноз шкіри і видимих слизових, тахікардія, диспноє. Рентгенологічно: в базальних відділах обох легень множинні дрібно- та середньовогнищеві тіні, збагачений, деформований по дрібнопе-тлистому типу, нечіткий легеневий малюнок. Загальний аналіз крові без змін. Який найбільш імовірний діагноз? On the 2nd day, a patient with multiple fractures of both lower extremities developed shortness of breath, cyanosis of the skin and visible mucous membranes, tachycardia, dyspnea. X-ray: in the basal parts of both lungs multiple small- and medium-focal shadows, enriched, deformed in the small-loop type, indistinct pulmonary pattern. The general blood test was not without changes. What is the most likely diagnosis?

Набряк легень Pulmonary edema

Двобічна дрібновогнищева пневмонія Bilateral small foci pneumonia

Гіпостатична пневмонія Hypostatic pneumonia

Тромбоемболія Thromboembolism

Респіраторний дистрес-синдром Respiratory distress syndrome

102 / 200
Хворому 70-ти років в стані клінічної смерті проводяться реанімаційні заходи. На ЕКГ фібриляція шлуночків. Які першочергові дії? A 70-year-old patient in a state of clinical death is being resuscitated. Ventricular fibrillation on the ECG. What are the primary actions?

Хімічна дефібриляція розчином калію хлориду Chemical defibrillation with potassium chloride solution

Антиаритмічні препарати Antiarrhythmic drugs

Діуретики Diuretics

Електрична дефібриляція Electrical defibrillation

Симпатоміметичні препарати Sympathomimetic drugs

103 / 200
Породілля 25-ти років їла горіх і несподівано поперхнулася, закашлялася; з’явилося утруднене дихання. При огляді виявлений періодичний кашель, подовжений видих, відставання правої половини грудної клітки при диханні. Аускультативно виявлено ослаблене дихання справа, рентгенологічно - ателектаз правої легені. Який метод лікування? A 25-year-old woman in labor ate a nut and suddenly snorted and coughed; difficulty breathing appeared. During the examination, a periodic cough, prolonged exhalation, lagging of the right half of the chest were found during breathing. Auscultation revealed weakened breathing on the right, x-ray - atelectasis of the right lung. What is the method of treatment?

Трахеотомія Tracheotomy

Торакотомія Thoracotomy

Бронхоскопія, видалення стороннього тіла Bronchoscopy, foreign body removal

Протинабрякові ендоларингеальні інгаляції Antiedema endolaryngeal inhalations

Езофагоскопія Esophagoscopy

104 / 200
Постраждалий в результаті ДТП чоловік 66-ти років непритомний. На голові численні садна та рани, дихання часте та поверхневе, ЧДР- 30/хв, АТ- 90/50 мм рт.ст., ЧСС- 120/хв., ціаноз шкірних покривів. Що слід застосувати під час транспортування хворого до лікарні? A 66-year-old man injured in a road accident is unconscious. There are numerous bruises and wounds on the head, breathing is frequent and shallow, blood pressure - 30/min, blood pressure - 90/ 50 mm Hg, heart rate - 120/min., cyanosis of the skin. What should be used when transporting the patient to the hospital?

Інтубація трахеї Tracheal intubation

Допоміжна вентиляція легень через маску Assisted lung ventilation through a mask

Введення дихальних аналептиків Introduction of respiratory analeptics

Інсуфляція зволоженого кисню Insufflation of humidified oxygen

Введення еуфіліну Introducing Euphilin

105 / 200
У дитини 3-х днів блювання з домішками крові, кал чорного кольору. Лабораторно: подовжений час згортання крові, гіпопротромбінемія, нормативна кількість тромбоцитів. Діагностовано геморагічну хворобу новонароджених. Який препарат оптимально використати в цій ситуації? The child has been vomiting for 3 days with blood impurities, black stool. Laboratory: prolonged blood clotting time, hypoprothrombinemia, normal platelet count. Diagnosed with hemorrhagic disease of newborns. What drug is optimal to use in this situation?

Вікасол Vikasol

Етамзилат натрію Etamsylate sodium

Аскорбінова кислота Ascorbic acid

Ілюконат кальцію Calcium Iluconate

Фібриноген Fibrinogen

106 / 200
Хлопчик 7-ми років хворіє третій день: підвищення температури тіла, біль у горлі, яскрава гіперемія слизової зіву, на гі-перемованій шкірі - крапчаста пурпурно- червона висипка, шкіра суха з нестійким білим дермографізмом. Який найбільш імовірний діагноз? A 7-year-old boy has been ill for the third day: fever, sore throat, bright hyperemia of the mucous membrane of the pharynx, a spotted purple-red rash , the skin is dry with unstable white dermographism. What is the most likely diagnosis?

Інфекційна еритема Erythema infectiosum

Краснуха Krasnukha

Скарлатина Scarlatina

Кір Measles

Псевдотуберкульоз Pseudotuberculosis

107 / 200
У дитини, хворої на грип, спостерігається втрата свідомості. При люмбальній пункції спинномозкова рідина витікала частими краплями, цитоз - 5 клітин. Про що в першу чергу треба думати? A child with flu has lost consciousness. During a lumbar puncture, cerebrospinal fluid leaked out in frequent drops, cytosis - 5 cells. What should you think about first?

Непритомність Fainting

Колапс Collapse

Менінгіт Meningitis

Нейротоксикоз Neurotoxicosis

Епілепсія Epilepsy

108 / 200
Хворий 59-ти років звернувся до лікаря із скаргами на пожовтіння шкірних покривів, свербіж. Пожовтів 4 дні тому після перенесеного больового нападу. Об’єктивно: шкірні покриви і склери жовтушні. На тілі наявні численні розчухи. У крові: білірубін - 218 мкмоль/л., пряма фракція -182 мкмоль/л, лужна фосфатаза - 4,2 мкмоль/год мл, білок - 68 г/л, АсАТ - 0,716, АлАТ - 0,813. Який провідний клінічний синдром у хворого? A 59-year-old patient turned to the doctor with complaints of yellowing of the skin, itching. He turned yellow 4 days ago after suffering a painful attack. Objectively: the skin and the sclera is yellow. There are numerous lacerations on the body. In the blood: bilirubin - 218 μmol/l, direct fraction - 182 μmol/l, alkaline phosphatase - 4.2 μmol/h ml, protein - 68 g/l, AsAT - 0.716, AlAT - 0.813. What is the leading clinical syndrome in the patient?

Печінково-клітинної недостатності Hepatocellular failure

Мезенхімально-запальний Mesenchymal-inflammatory

Холестазу Cholestasis

Астено-невротичний Astheno-neurotic

Портальної гіпертензії Portal hypertension

109 / 200
У хворої 26-ти років грип, важкий перебіг. На тлі гіпертермії у неї з’явились туга, плаксивість, намагання втекти, говорила, що її зачарували, що вона вмирає, просила пробачення, відмовлялась від їжі. Потім стала дезорієнтованою, збудженою, відповіді короткі, безбарвні, мова - незв’язна; чинила опір огляду. Як розцінити стан хворої? A 26-year-old patient has the flu, a severe course. Against the background of hyperthermia, she developed anxiety, tearfulness, attempts to escape, said that she was bewitched, that she was dying, asked for forgiveness, refused food. Then she became disoriented, agitated, her answers were short, colorless, her speech was incoherent; she resisted examination. How to assess the patient's condition?

Депресивний стан Depressive state

Діенцефальний синдром Diencephalic syndrome

Дебют шизофренії Schizophrenia onset

Інфекційний психоз Infectious psychosis

Нейроциркуляторна дистонія Neurocirculatory dystonia

110 / 200
Хворий 41-го року, який знаходився під наглядом впродовж 3-х років з приводу пароксизмальної шлуночкової тахікардії, перебуваючи на робочому місці, втратив свідомість. Була зафіксована відсутність серцевої діяльності, пульсації на сонних артеріях. Дихання відсутнє, зіниці широкі без реакції на світло. Чим зумовлена раптова смерть хворого? A 41-year-old patient, who was under observation for 3 years due to paroxysmal ventricular tachycardia, lost consciousness while at work. The absence of cardiac activity, pulsations on the carotid arteries. There is no breathing, the pupils are wide without reaction to light. What caused the patient's sudden death?

Гостре порушення мозкового кровообігу Acute cerebrovascular accident

Нестабільна стенокардія Unstable angina

Гострий інфаркт міокарда Acute myocardial infarction

Тромбоемболія легеневої артерії Thromboembolism of the pulmonary artery

Фібриляція шлуночків Ventricular fibrillation

111 / 200
У хворого на 5-й день хвороби, що почалася гостро з підвищення температури тіла до 40oC, сильного головного болю та збудження, з’явилася екзантема розеольознопетехіального характеру, яка локалізується переважно на бокових поверхнях тулуба та згинальних поверхнях кінцівок. Які лабораторні дослідження необхідно провести для підтвердження діагнозу у хворого? On the 5th day of the illness, which began acutely with an increase in body temperature to 40oC, severe headache and excitement, the patient developed a roseolose-petechial exanthema, which it is localized mainly on the lateral surfaces of the trunk and flexing surfaces of the limbs. What laboratory tests should be performed to confirm the diagnosis in the patient?

Гемокультура Hemoculture

Клінічний аналіз сечі Clinical urinalysis

Серологічні (РАР, РЗК, РНГА) Serological (PAR, RZK, RNHA)

Імунофлюоресцентні Immunofluorescent

Бактеріологічне дослідження калу Bacterial examination of feces

112 / 200
70- річного хворого на нестабільну стенокардію з серцевою недостатність II А ст. госпіталізовано у відділення інтенсивної терапії. Об’єктивно: набряки нижніх кінцівок, АТ- 140/90 мм рт.ст., ЧСС- 98/хв. Яку дозу пропранололу доцільно призначити хворому? A 70-year-old patient with unstable angina pectoris with II A stage heart failure was hospitalized in the intensive care unit. Objectively: swelling of the lower extremities, blood pressure 140/90 mm Hg, heart rate - 98/min. What dose of propranolol should be prescribed to the patient?

100 мг на прийом 100 mg per dose

120 мг на прийом 120 mg per dose

1 мг на прийом 1 mg per dose

60 мг на прийом 60 mg per dose

20 мг на прийом 20 mg per dose

113 / 200
Пацієнту 86-ти років з декомпенсованою формою серцево-легеневої недостатності, після падіння на правий бік встановлено діагноз: перелом шийки правої стегнової кістки. Яке лікування слід запропонувати? An 86-year-old patient with a decompensated form of cardiopulmonary failure, after falling on the right side, was diagnosed with a fracture of the neck of the right femur. What treatment should be offered?

Оперативне (остеосинтез шийки кутопо-дібною пластинкою із гвинтами) Operative (osteosynthesis of the neck with an angular plate with screws)

Скелетним витягом By skeletal extract

Оперативне (остеосинтез шийки спонгі-озними гвинтами) Operative (neck osteosynthesis with cancellous screws)

Ендопротезування кульшового суглоба Hip arthroplasty

Консервативне (деротаційний 'чобі-ток'та симптоматична терапія) Conservative (derotational 'chobi-talk' and symptomatic therapy)

114 / 200
У жінки 27-ми років рана середньої третини лівої литки 5х3 см, неправильної форми, з рваними краями та помірною кровотечею. Близько 15 хвилин тому її покусав бродячий пес. Яка перша допомога? A 27-year-old woman has a 5x3 cm wound in the middle third of her left calf, irregular in shape, with ragged edges and moderate bleeding. She was bitten by a stray dog ​​about 15 minutes ago. What is the first aid?

ПХО. Рану зашити. Госпіталізація в хірургічний стаціонар для проведення антирабічної профілактики PHO. Suture the wound. Hospitalization in a surgical hospital for anti-rabies prophylaxis

ПХО. Рану не зашивати. Госпіталізація в хірургічний стаціонар для проведення антирабічної профілактики PHO. The wound should not be sutured. Hospitalization in a surgical hospital for anti-rabies prophylaxis

ПХО. Рану не зашивати. Амбулаторне спостереження хірурга та рабіолога PHO. The wound should not be sutured. Outpatient observation by a surgeon and a rabiologist

ПХО. Рану зашити. Амбулаторне спостереження хірурга та рабіолога PHO. Suture the wound. Outpatient observation by a surgeon and a rabiologist

- -

115 / 200
В стаціонар госпіталізовано дитину 11-ти місяців на 2-й день хвороби з підвищенням температури до 38oC, багаторазовим блюванням і частими водянистими випорожненнями. Маса тіла знижена на 9%. Який метод регідратації показаний у даному випадку? An 11-month-old child was hospitalized on the 2nd day of illness with a temperature rise to 38oC, repeated vomiting and frequent watery stools. Body weight decreased by 9% . What method of rehydration is indicated in this case?

Внутрішньовенний крапельний Intravenous drip

Введення рідини ендогастрально Endogastric fluid administration

Оральна регідратація Oral rehydration

Внутрішньовенний струминний Intravenous jet

Введення рідини ректально крапельно Fluid administration by rectal drip

116 / 200
Під час серцево-легеневої реанімації без венозного доступу можна вводити ендотрахеально: During cardiopulmonary resuscitation without venous access, it is possible to enter endotracheally:

Натрію гідрокарбонат Sodium bicarbonate

Дофамін Dopamine

Кальцію глюконат Calcium gluconate

Адреналін Adrenaline

Кальцію хлорид Calcium chloride

117 / 200
Хворий після ін’єкції бензогексонію при вставанні з ліжка відчув запаморочення та нудоту. Об’єктивно: шкірні покриви бліді, теплі. АТ- 70/30 мм рт.ст., пульс слабкого наповнення, тахікардія більше 120/хв. Тони серця значно приглушені. Тахіпное. Живіт м’який, не болючий. В якому положенні хворого слід надавати невідкладну допомогу? After an injection of benzohexonium, the patient felt dizzy and nauseous when getting out of bed. Objectively: the skin is pale, warm. Blood pressure - 70/30 mm Hg. st., weakly filling pulse, tachycardia more than 120/min. Heart sounds are significantly muffled. Tachypnea. Abdomen is soft, not painful. In what position of the patient should emergency care be provided?

У положенні Тренделенбурга In Trendelenburg position

У горизонтальному положенні на животі In a horizontal position on the stomach

У положенні Фовлера In Fowler's position

У горизонтальному положенні на правому боці In a horizontal position on the right side

У тому положенні, у якому знаходиться хворий In the position in which the patient is

118 / 200
Хворого госпіталізовано в інфекційне відділення зі скаргами на двоїння предметів, 'сітку'перед очима, слабкість, запаморочення, сухість у роті, нудоту та тяжкість в епігастрії. Об’єктивно: стан середньої важкості, млявий, температура тіла 36,2oC, птоз, мідріаз, анізокорія, дихання везикулярне, ЧД- 20/хв., Р8- 88/хв., АТ-110/70 мм рт.ст. За добу до захворювання пив алкоголь, їв в’ялену рибу, кров’яну ковбасу. З чого слід почати лікування? The patient was hospitalized in the infectious department with complaints of double objects, a 'net' in front of the eyes, weakness, dizziness, dry mouth, nausea and heaviness in the epigastrium. About' Objectively: moderate condition, lethargic, body temperature 36.2oC, ptosis, mydriasis, anisocoria, vesicular breathing, BH-20/min., P8-88/min., BP-110/70 mm Hg. The day before disease, drank alcohol, ate dried fish, blood sausage. Where should treatment begin?

Промивання шлунка Gastric lavage

Гемосорбція Hemosorption

Специфічний антидот Specific antidote

Антибіотики Antibiotics

Специфічна сироватка Specific Serum

119 / 200
Постраждалий доставлений на МПП з вогнища через 3 години після застосування хімічної зброї. Скаржиться на наростаючу задишку, кашель з великою кількістю харкотиння. Хворий збуджений, намагається зайняти сидяче положення. Шкірні покриви синюшні, під час кашлю пінисте рожеве харкотиння. ЧД- 32/хв. Дихання ослаблене, у підлопаткових ділянках - вологі різнокаліберні хрипи. Яка отруйна речовина може викликати такі скарги? The victim was brought to the emergency room from the fire 3 hours after the use of chemical weapons. He complains of increasing shortness of breath, cough with a lot of sputum. The patient is agitated, trying to take a sitting position. The skin is bluish, when coughing, foamy pink sputum is produced. BH - 32/min. Breathing is weakened, in the subscapular areas - wet, various-caliber rales. What poisonous substance can cause such complaints?

Хлорпікрин Chlorpicrin

Фосген Phosgene

Люїзит Lewisite

Зарин Sarin

Іприт Mustard

120 / 200
Група робітників виробництва по виготовленню анілінових барвників доставлена у лікувальний заклад після аварії зі скаргами на загальну слабкість, головний біль, біль у епігастрії і блювання. Виявлено гепатоспленомегалію, анемію, ретикуло-цитоз, тільця Гейнца-Ерліха в еритроцитах, підвищення білірубіну. Яке дослідження дозволить підтвердити факт отруєння аніліновими барвниками? A group of workers in the production of aniline dyes was taken to a medical institution after the accident with complaints of general weakness, headache, pain in the epigastrium and vomiting. Hepatosplenomegaly, anemia, reticulocytosis, Heinz-Ehrlich bodies in erythrocytes, increased bilirubin. What research will confirm the fact of poisoning with aniline dyes?

Сечовина Urea

Рівень карбоксигемоглобіну Carboxyhemoglobin level

Креатинін Creatinine

Рівень метгемоглобіну Methemoglobin level

Фетальний гемоглобін Fetal hemoglobin

121 / 200
Чоловік 72-х років, що знаходиться у відділенні хірургії з приводу тромбозу глибоких вен гомілки, знепритомнів. Під час огляду: ціаноз верхньої половини тулуба і обличчя, дихання відсутнє, пульс на сонних артеріях ниткоподібний. Про яке ускладнення слід думати? A 72-year-old man, who is in the surgery department due to thrombosis of the deep veins of the leg, fainted. During the examination: cyanosis of the upper half of the body and face, there is no breathing , the pulse on the carotid arteries is filiform. What complication should be considered?

Гостра лівошлуночкова недостатність Acute left ventricular failure

Фібриляція шлуночків Ventricular fibrillation

Асистолія Asystole

Гострий інфаркт міокарда Acute myocardial infarction

ТЕЛА BODY

122 / 200
Дівчинка 12-ти років доставлена до гінекологічного відділення із рясною кровотечею зі статевих шляхів впродовж 24-х годин. Об’єктивно: блідість шкіри та слизових оболонок. При гінекологічному обстеженні підтверджено наявність маткової кровотечі, матка дещо збільшена, щільна. Додатки не визначаються, їх область безболісна. У крові: Hb- 70 г/л. Яка тактика лікаря? A 12-year-old girl was brought to the gynecological department with profuse bleeding from the genital tract for 24 hours. Objectively: paleness of the skin and mucous membranes. At the gynecological the examination confirmed the presence of uterine bleeding, the uterus is slightly enlarged, dense. The appendages are not identified, their area is painless. In the blood: Hb - 70 g/l. What are the doctor's tactics?

Знеболююча терапія Pain therapy

Гормональний гемостаз та антианемічна терапія Hormonal hemostasis and anti-anemic therapy

Антианемічна терапія Antianemic therapy

Оперативне лікування Operative treatment

Протизапальна терапія Anti-inflammatory therapy

123 / 200
Хвора 20 років, у шлюбі 2 роки. Статеве життя регулярне, від вагітності не за-побігалась, остання менструація - 6 тижнів тому. По дорозі на роботу виник біль у низу живота, втрата свідомості. Швидкою допомогою доставлена в гінекологічне відділення у важкому стані: шкіра і слизові оболонки бліді, АТ- 80/40 мм рт.ст., Ps- 112/хв. Який імовірний діагноз? The patient is 20 years old, has been married for 2 years. Sex life is regular, pregnancy has not been delayed, the last menstruation was 6 weeks ago. On the way to work, pain arose in the lower abdomen, loss of consciousness. She was taken by ambulance to the gynecology department in critical condition: skin and mucous membranes are pale, blood pressure - 80/40 mm Hg, Ps - 112/min. What is the probable diagnosis?

Апоплексія яєчника. Больовий шок Apoplexy of the ovary. Painful shock

Позаматкова вагітність. Геморагічний шок Ectopic pregnancy. Hemorrhagic shock

Кардіогенний шок Cardiogenic shock

Септичний шок Septic shock

Тромбоемболія легеневої артерії Thromboembolism of the pulmonary artery

124 / 200
У першороділлі народився хлопчик масою 3000 г у глибокій асфіксії. Після відсмоктування слизу, вентиляції легень і непрямого масажу серця лікар вирішив застосувати адреналін. Яким методом краще його ввести? A boy weighing 3,000 g was born in the first delivery in deep asphyxia. After suctioning the mucus, ventilating the lungs and indirect heart massage, the doctor decided to apply adrenaline. What is the best method to inject it?

Внутрішньовенно Intravenous

У м’яз серця In the heart muscle

Ендотрахеально Endotracheal

Внутрішньом’язово Intramuscular

Підшкірно Subcutaneous

125 / 200
У спекотний літний день (температура повітря у тіні 35oC) група людей відпочивала біля ставка. Молоду жінку укусила гадюка степова у нижню третину гомілки. Невідкладна допомога передбачає: On a hot summer day (air temperature in the shade 35oC), a group of people rested near a pond. A young woman was bitten by a steppe viper in the lower third of the leg. Emergency care involves:

Протерти місце укусу вологою тканиною, накласти джгут вище місця укусу, негайне транспортувати до лікарні Wipe the bite site with a wet cloth, apply a tourniquet above the bite site, immediately transport to the hospital

Провести іммобілізацію кінцівки, негайне транспортувати до лікарні Immobilize the limb, immediately transport to the hospital

Накласти вологу тканинну серветку, дати випити 100 мл горілки, транспортувати до лікарні Apply a wet tissue, give 100 ml of vodka to drink, transport to hospital

Протерти місце укусу вологою тканиною, провести іммобілізацію кінцівки, негайне транспортувати до лікарні Wipe the bite site with a wet cloth, immobilize the limb, immediately transport to the hospital

Припекти місце укусу, провести іммобілізацію кінцівки, негайне транспортувати до лікарні Cauterize the bite site, immobilize the limb, immediately transport to the hospital

126 / 200
Хвору 40-ка років госпіталізовано до реанімаційного відділення в тяжкому стані. При огляді: акроціаноз, бронхорея, та-хіпное, АТ- 110/60 мм рт.ст., Ps- 52/хв. Зі слів родичів жінка 5 годин тому проводила обробку насаджень картоплі. Яка група речовин стала причиною отруєння? A 40-year-old patient was hospitalized in the intensive care unit in a serious condition. On examination: acrocyanosis, bronchorrhoea, ta-hypnoea, blood pressure - 110/60 mm Hg ., Ps- 52/min. According to relatives, the woman was processing potato plantations 5 hours ago. What group of substances caused the poisoning?

Отруєння чадним газом Monoxide poisoning

Отруєння кислотами Acid poisoning

Отруєння хлорорганічними сполуками Poisoning by organochlorine compounds

Отруєння фосфорорганічними сполуками Poisoning with organophosphorus compounds

Отруєння важкими металами Heavy metal poisoning

127 / 200
В клініку звернулось троє молодих людей зі скаргами на нудоту, блювання, спрагу, головний біль, біль в ногах, погіршення зору, миготіння 'мушок'перед очима. Напередодні вживали алкогольні напої. Лікар запідозрив отруєння метиловим спиртом. Яких заходів необхідно вжити для правильного надання медичної допомоги? Three young people came to the clinic with complaints of nausea, vomiting, thirst, headache, pain in the legs, impaired vision, flickering of 'flies' in front of the eyes. The day before consumed alcoholic beverages. The doctor suspected methyl alcohol poisoning. What measures should be taken to properly provide medical assistance?

Промивання шлунка, сольові послаблюючі, введення антидоту (етиловий спирт), термінова госпіталізація Gastric lavage, saline laxatives, administration of an antidote (ethyl alcohol), urgent hospitalization

Промивання шлунка, сольові послаблюючі, консультація офтальмолога, амбулаторне спостереження Gastric lavage, saline laxatives, ophthalmologist consultation, outpatient follow-up

Термінова госпіталізація в реанімаційне відділення Urgent hospitalization in the intensive care unit

Промивання шлунка, сольові послаблюючі, термінова госпіталізація в інфекційне відділення Gastric lavage, saline laxatives, urgent hospitalization in the infectious department

Активоване вугілля, форсований діурез Activated charcoal, forced diuresis

128 / 200
Чоловік випав з кузова вантажівки, вниз головою. Загальмований. При голосному звертанні відкриває очі. Руки та ноги звисають. Дихає важко. Пульс прискорений. Які дії лікаря швидкої допомоги? The man fell from the truck body, upside down. He slowed down. He opened his eyes when called loudly. His arms and legs dangled. He was breathing hard. His pulse was fast. What actions did the emergency doctor take? ?

Термінова госпіталізація до нейрохірургічного відділення Urgent hospitalization in the neurosurgery department

Іммобілізація хребта, повітровід, термінова госпіталізація Spinal immobilization, airway, urgent hospitalization

Седативні засоби, еуфілін внутрішньовенно Sedatives, Euphilin intravenously

Розпочати штучну вентиляцію легень Start CPR

Дочекатися прибуття міліції та госпіталізувати потерпілого Wait for the police to arrive and hospitalize the victim

129 / 200
У хворого внаслідок автомобільної катастрофи проникаюче поранення грудної клітки. При цьому є зяюча рана, через яку плевральна порожнина сполучається із зовнішнім середовищем. Загальний стан хворого важкий. Виражена тахікардія, задишка, ціаноз шкірних покривів і слизових оболонок. При аускультації дихальні шуми на боці поранення не прослуховуються, при перкусії - звук з коробковим відтінком. Який найбільш імовірний діагноз? The patient has a penetrating wound to the chest as a result of a car accident. There is also a gaping wound through which the pleural cavity communicates with the external environment. The general condition of the patient is serious. Pronounced tachycardia, shortness of breath, cyanosis of the skin and mucous membranes. During auscultation, breathing noises on the side of the wound are not heard, during percussion - a sound with a box tone. What is the most likely diagnosis?

Емфізема середостіння Emphysema of the mediastinum

Закритий пневмоторакс Closed pneumothorax

Клапанний (напружений) пневмоторакс Valvular (tension) pneumothorax

Відкритий пневмоторакс Open pneumothorax

Гемоторакс Hemothorax

130 / 200
Хлопчик 12-ти років контактний з батьком, хворим на легеневу форму чуми. Якими препаратами слід провести дитині профілактичне лікування? A 12-year-old boy is in contact with his father, who is sick with the pneumonic form of the plague. What drugs should be given to the child for preventive treatment?

Імуноглобулін або анатоксин Immunoglobulin or toxoid

Стрептоміцин або доксициклін Streptomycin or doxycycline

Гентаміцин або нетроміцин Gentamicin or Netromycin

Пєніцилін або цефотаксим Penicillin or Cefotaxime

Бісептол або сульфодімезин Biseptol or sulfodimesin

131 / 200
Чоловік 40-ка років знепритомнів прямо на вулиці. Дихання не визначається, пульс на сонних артеріях відсутній. З чого необхідно почати реанімаційні заходи? A 40-year-old man fainted right on the street. Breathing is undetectable, there is no pulse on the carotid arteries. Where should resuscitation measures be started?

Дихання 'рот в ніс' Mouth-to-nose breathing

Дихання 'рот у рот' Mouth-to-mouth breathing

Непрямий масаж серця Indirect heart massage

Огляд зіниць з метою виявлення мідріазу Pupillary examination for mydriasis

Забезпечення прохідності дихальних шляхів Ensuring airway patency

132 / 200
Чоловіку 40-ка років на будівництві балкою, що впала, перетиснуло обидві нижні кінцівки. На усунення балки знадобилося більше години. Яку допомогу необхідно надати даному пацієнту? A 40-year-old man was crushed by a falling beam on a construction site. It took more than an hour to remove the beam. What help should be given to this patient?

Проведення форсованого діурезу Performing forced diuresis

Знеболювання Pain relief

Усе перераховане All of the above

Туге бинтування кінцівки Tight bandaging of the limb

Іммобілізація кінцівки Immobilization of the limb

133 / 200
У пацієнта під час операції під загальним знеболенням на органах черевної порожнини відбулася зупинка кровообігу. В даній ситуації: During an operation under general anesthesia, blood circulation stopped in the organs of the abdominal cavity. In this situation:

Масаж серця не проводиться, достатньо обмежитися штучною вентиляцією легень Heart massage is not performed, it is enough to limit yourself to artificial lung ventilation

- -

Необхідне проведення прямого масажу серця через діафрагму Direct heart massage through the diaphragm is necessary

Необхідний розтин грудної клітки і проведення прямого масажу серця Chest dissection and direct heart massage is necessary

Необхідне проведення непрямого масажу серця Indirect cardiac massage is required

134 / 200
У пацієнта з інкурабельною онкологічною патологією діагностована клінічна смерть. Який обсяг екстреної допомоги має бути наданий? A patient with incurable oncological pathology is diagnosed with clinical death. What amount of emergency care should be provided?

Непрямий масаж серця Indirect heart massage

Дефібриляція Defibrillation

Допомога не надається No help

Забезпечити прохідність дихальних шляхів Ensure patency of respiratory tract

Штучна вентиляція легень Artificial lung ventilation

135 / 200
Хворий 8 годин тому прийняв 10 мг фенобарбіталу. На тлі коми розвинувся короткочасний судомний припадок, після чого міоз змінився мідріазом, з’явилися менінге-альні знаки, гіпертермія до 39oC. Визначити причину судомного припадку: The patient took 10 mg of phenobarbital 8 hours ago. On the background of a coma, a short-term seizure developed, after which miosis was replaced by mydriasis, meningeal signs appeared, hyperthermia to 39oC. Determine the cause of a convulsive attack:

Гіпоксія і набряк головного мозку Hypoxia and cerebral edema

Інфаркт мозку Cerebral infarction

Епілептичний напад Epileptic attack

Менінгіт Meningitis

Субарахноїдальний крововилив Subarachnoid hemorrhage

136 / 200
Під час зимової риболовлі один з рибалок провалився під лід. Рятувальники протягом 20 хвилин витягли потерпілого. Самостійне дихання і свідомість відсутні. Пульс на сонних артеріях не визначається, зіниці розширені, мармуровість шкірних покривів. Як слід розцінити стан потерпілого і доцільність реанімаційних заходів? During winter fishing, one of the fishermen fell under the ice. The rescuers pulled out the victim within 20 minutes. There is no independent breathing and consciousness. The pulse on the carotid arteries is not determined, the pupils are dilated , marbling of the skin. How should the condition of the victim and the feasibility of resuscitation measures be assessed?

Клінічна смерть, реанімаційні заходи необхідні Clinical death, resuscitation measures are necessary

Соціальна смерть, реанімаційні заходи не показані Social death, resuscitation measures not shown

Зупинка кровообігу, реанімаційні заходи необхідні Closing stop, resuscitation measures are necessary

Біологічна смерть, реанімаційні заходи не показані Biological death, resuscitation measures not shown

Смерть мозку, реанімаційні заходи не показані Brain death, resuscitation measures not shown

137 / 200
Рядового через 2 години після вибуху на АЕС доставлено у МПП. Через 45 хвилин після опромінення з’явилися слабкість, нудота, багаторазове блювання, збудження, запаморочення. Після вибуху прийняв усередину 2 таблетки етаперазину. При обстежені - виражена гіперемія шкіри, температура тіла - 37,4oC. Дихання везикулярне, тони серця звучні, ЧСС- 128/хв., АТ-100/60 мм рт.ст. Показник індивідуального дозиметра - 4,8 Гр. Які дослідження дозволять підтвердити гостру променеву хворобу у потерпілого? 2 hours after the explosion at the nuclear power plant, the private was taken to the emergency department. 45 minutes after exposure, weakness, nausea, repeated vomiting, agitation, dizziness appeared. After the explosion took 2 tablets of Etaperazine internally. When examined, there was marked hyperemia of the skin, body temperature - 37.4oC. Breathing is vesicular, heart sounds are sonorous, heart rate - 128/min., BP - 100/60 mm Hg. Individual dosimeter indicator - 4, 8 Gy. What studies will confirm acute radiation sickness in the victim?

Визначення кількості лейкоцитів, тромбоцитів, швидкості зсідання еритроцитів Determining the number of leukocytes, platelets, erythrocyte sedimentation rate

Визначення рівня катехоламінів сечі Determining the level of urinary catecholamines

Визначення рівня глікемії, амілази крові Determining the level of blood glucose, amylase

Визначення загального холестерину, креатиніну Determination of total cholesterol, creatinine

Визначення рівня бета-ліпопротеїдів, тригліцеридів Determining the level of beta-lipoproteins, triglycerides

138 / 200
Першовагітна у терміні 29 тижнів поскаржилась на біль у правій нижній кінцівці. Об’єктивно: розміри правої нижньої кінцівки на рівні гомілки - більші лівої нижньої кінцівки на 2-3 см, колір шкірних покривів без змін, але відмічається помірна пастозність правої стопи; болючості м’язів нижніх кінцівок не відмічається. Який найбільш імовірний діагноз? A first-time pregnant woman at 29 weeks complained of pain in the right lower limb. Objectively: the size of the right lower limb at the level of the shin is 2-3 larger than the left lower limb cm, the color of the skin is unchanged, but there is moderate pastiness of the right foot; there is no pain in the muscles of the lower extremities. What is the most likely diagnosis?

Пізній гестоз вагітних Late gestosis of pregnant women

Набряки вагітних Swelling of pregnant women

Варикозне розширення вен Varicose veins

Флеботромбоз глибоких вен правої нижньої кінцівки Phlebothrombosis of the deep veins of the right lower limb

Загроза передчасних пологів Threat of premature birth

139 / 200
Внаслідок передчасних пологів, при терміні гестації 33-34 тижні, народилася дитина масою 2000 г у важкій асфіксії. Дихання відсутнє, серцебиття до 90/хв. Тактика лікаря: As a result of premature birth, at a gestation period of 33-34 weeks, a child weighing 2000 g was born in severe asphyxia. There is no breathing, heartbeat up to 90/min. Doctor's tactics:

Відсмоктати слиз із верхніх дихальних шляхів Suction mucus from the upper respiratory tract

Помістити в кювез Put in cuvez

Провести інтубацію трахеї новонародженого Intubate a newborn's trachea

Провести тактильне подразнення Perform tactile stimulation

Ввести адреналін внутрішньосерцево Inject epinephrine intracardiacally

140 / 200
Хвора 47-ми років скаржиться на головний біль, запаморочення, 'миготіння мушок'перед очима, біль у ділянці серця. Стан погіршився гостро після психоемоційного навантаження. Об’єктивно: тони серця ритмічні, звучні. ЧСС- 105/хв., АТ-220/110 мм рт.ст. Межі серця розширені вліво. Живіт м’який, неболючий. Набряки відсутні. Який найбільш імовірний діагноз? A 47-year-old patient complains of headache, dizziness, 'flickering of flies' before the eyes, pain in the heart area. The condition worsened sharply after psycho-emotional stress. About' objectively: heart sounds are rhythmic, sonorous. Heart rate - 105/min., blood pressure - 220/110 mm Hg. The borders of the heart are expanded to the left. The abdomen is soft, painless. There is no swelling. What is the most likely diagnosis?

Діенцефальний криз Diencephalic crisis

Синусова тахікардія Sinus tachycardia

Реакція на стрес Reaction to stress

Клімакс Climax

Гіпертензивний криз Hypertensive crisis

141 / 200
В медичний пункт полку з вогнища радіаційної катастрофи доставлено 15 потерпілих. Через 20-25 хвилин після вибуху стали відчувати різку слабкість, запаморочення, нудоту, з’явилось невгамовне блювання. Стан хворих тяжкий. Кволі, апатичні, на запитання відповідають з великою затримкою. ЧСС- 120/хв. з частими екстра-систолами, АТ- 70/30 мм рт.ст., ЧД- 28/хв. За даними індивідуального дозиметра доза отриманого випромінювання склала 8 Гр. Який найбільш імовірний діагноз? 15 victims were brought to the regimental medical center from the site of the radiation disaster. 20-25 minutes after the explosion, they began to feel sharp weakness, dizziness, nausea, and uncontrollable vomiting appeared The condition of the patients is serious. Weak, apathetic, they answer questions with a long delay. Heart rate - 120/min with frequent extra-systoles, blood pressure - 70/30 mm Hg, BP - 28/min. According to the data of the individual dosimeter, the dose received radiation was 8 Gy. What is the most likely diagnosis?

Гостра променева хвороба II ст. Acute radiation sickness of the II century.

Гостра променева хвороба III ст. Acute radiation sickness of the III century.

Гостра променева хвороба I ст. Acute radiation sickness of the 1st century

Гостра променева хвороба IV ст. Acute radiation sickness of the IV century.

Гостра променева хвороба V ст. Acute radiation sickness of the 5th century.

142 / 200
У хворого під час переливання консервованої крові (через 5 хвилин) виникли: холодний піт, утруднення дихання. АТ- 70/50 мм рт.ст., Ps- 100/хв, аритмічний. Яке ускладнення має місце в даному випадку? During the transfusion of canned blood (after 5 minutes), the patient developed: cold sweat, difficulty breathing. BP- 70/50 mm Hg, Ps- 100/min, arrhythmic. What complication occurs in this case?

Напад бронхіальної астми Bronchial asthma attack

Колапс Collapse

Гострий інфаркт міокарда Acute myocardial infarction

Загострення ХОЗЛ Aggravation of COPD

Анафілактична гемотрансфузійна реакція, шок Anaphylactic blood transfusion reaction, shock

143 / 200
У хворого на абсцес легенів 47-ми років після введення тієнаму розвинувся анафілактичний шок. Який препарат треба ввести хворому в першу чергу? A 47-year-old patient with a lung abscess developed anaphylactic shock after the introduction of tienam. Which drug should be administered to the patient first?

Допамін Dopamine

Супрастин Suprastin

Адреналін Adrenaline

Дексаметазон Dexamethasone

Еуфілін Euphilin

144 / 200
Підліток під час катання на ковзанах послизнувся та впав, забив голову, на декілька хвилин знепритомнів. Після опри-томнення скаржиться на помірний головний біль. Наявна ретроградна амнезія, вогнищної неврологічної симптоматики немає. Оберіть вірну тактику: While skating, the teenager slipped and fell, hit his head, and fainted for a few minutes. After waking up, he complains of a moderate headache. There is retrograde amnesia, focal there are no neurological symptoms. Choose the correct tactics:

Госпіталізувати хворого для динамічного спостереження до нейрохірургічного відділення Hospitalize the patient for dynamic observation to the neurosurgery department

Порекомендувати постраждалому обмеження фізичних навантажень та анальгетики Recommend to the victim the restriction of physical activity and analgesics

Призначити препарати, що поліпшують мозковий кровообіг Prescribe drugs that improve cerebral blood circulation

Призначити помірну дегідратацію та анальгетики Prescribe moderate dehydration and analgesics

Призначити анальгетики Prescribe analgesics

145 / 200
У дитини 7-ми років, що знаходиться на ШВЛ, діагностовано напружений пневмоторакс. Які лікувальні заходи треба вжити в першу чергу? A 7-year-old child who is on a mechanical ventilator has been diagnosed with tension pneumothorax. What medical measures should be taken first?

Перевести хворого на ШВЛ Transfer patient to ventilator

Призначити інгаляцію кисню Prescribe oxygen inhalation

Призначити бронхолітичні засоби Prescribe bronchodilator

Дренування плевральної порожнини за Бюлау Drainage of the pleural cavity according to Bülau

Використати метод дихання з постійним позитивним тиском (СДППТ) Use the CPAP method

146 / 200
6- ти місячна дитина госпіталізована з приводу 5-ти разового блювання, рідких випорожнень більше 10 разів на добу. Об’єктивно: неспокійна, риси обличчя загострені, шкіра та слизові оболонки бліді та сухі, велике тім’ячко запале, температура тіла 38, 7oC, ЧСС- 162/хв., тони серця глухі, живіт здутий, діурез зменшений, дефіцит маси -10%. У крові: Ht- 50%, К+- 3,4 ммоль/л; Na+-154 ммоль/л. Які першочергові заходи? A 6-month-old child was hospitalized for vomiting 5 times, loose stools more than 10 times a day. Objectively: restless, facial features sharpened, skin and mucous membranes are pale and dry, the big head is inflamed, body temperature is 38.7oC, heart rate is 162/min, heart sounds are dull, the stomach is distended, diuresis is reduced, mass deficit -10%. In the blood: Ht- 50%, K+- 3.4 mmol/l; Na+-154 mmol/l. What are the primary measures?

Призначення протиблювотних засобів Prescription of antiemetics

Призначення антибактеріальних препаратів Prescription of antibacterial drugs

Призначення жарознижуючих препаратів Prescription of antipyretic drugs

Введення глюкозо-сольових розчинів 3:1 Introduction of glucose-salt solutions 3:1

Введення глюкозо-сольових розчинів 2:1 Introduction of glucose-salt solutions 2:1

147 / 200
В реанімаційному відділенні у дитини з зупинкою серця після проведення інтуба-ції та ШВЛ почервоніли шкірні покриви, пульс на великих судинах, ЧСС- 40/хв., АТ-50/30 мм рт.ст. Яка подальша тактика? In the intensive care unit, a child with cardiac arrest after intubation and mechanical ventilation had reddened skin, pulse on large vessels, heart rate - 40/min., BP - 50/30 mm Hg. What are the next tactics?

Введення розчину допаміну Injection of dopamine solution

Введення 0,1% розчину атропіну Introduction of 0.1% atropine solution

Введення сольових розчинів Introduction of saline solutions

Введення серцевих глікозидів Introduction of cardiac glycosides

Введення глюкокортикоїдів Introduction of glucocorticoids

148 / 200
Вагітна в 35 тижнів надійшла зі скаргами на головний біль, нудоту, порушення зору. Об’єктивно: Ps- 104/хв., АТ- 170/120 мм рт.ст та 180/120 мм рт.ст., to- 36, 7oC. Генералізовані набряки. Сеча мутна, в малій кількості. Під час огляду у жінки з’явились дрібні посмикування м’язів обличчя та верхніх кінцівок. Потім тіло напружилось, вигнулось, припинилось дихання, обличчя стало синім, свідомість відсутня. Які реанімаційні заходи мають бути проведені першочергово? A 35-week pregnant woman came in with complaints of headache, nausea, visual disturbances. Objectively: Ps- 104/min., BP- 170/120 mm Hg and 180/120 mm Hg, to- 36, 7oC. Generalized edema. Urine cloudy, in small quantities. During the examination, the woman had small twitching of the muscles of the face and upper limbs. Then the body tensed , bent over, breathing stopped, the face turned blue, consciousness is absent. What resuscitation measures should be carried out as a priority?

Внутрішньом’язове введення сульфату магнію Intramuscular administration of magnesium sulfate

Дихання через маску впродовж 2-х годин Breathing through a mask for 2 hours

Внутрішньовенне введення гіпотензивних препаратів Intravenous administration of hypotensive drugs

Вичікувальна тактика Waiting tactics

ШВЛ, внутрішньовенне введення сульфату магнію Ventilation, intravenous administration of magnesium sulfate

149 / 200
Першенароджуюча доставлена зі скаргами на перейми впродовж 6 годин. Ps-84/хв., АТ-120/80 мм рт.ст. PV: шийка матки згладжена, відкриття 4 см, плідний міхур вкрай напружений. Під час огляду розірвались навколоплідні оболонки, вилилось 3,5 л світлих навколоплідних вод. Раптом з’явились ядуха, різкий біль за грудниною, ціаноз обличчя. Ps- 110/хв., ниткоподібний, АТ- 65/10 мм рт.ст. В чому полягає невідкладна медична допомога? A primiparous woman was delivered with complaints of contractions for 6 hours. Ps-84/min., BP-120/80 mm Hg. PV: cervix smoothed, the opening is 4 cm, the amniotic sac is extremely tense. During the examination, the amniotic membranes ruptured, 3.5 liters of light-colored amniotic fluid spilled out. Sudden dysentery, sharp pain behind the sternum, cyanosis of the face. Ps- 110/min., filamentous, BP- 65/10 mm Hg What is emergency medical care?

Введення спазмолітиків Introduction of antispasmodics

Надати тілу підвищене положення, інгаляції кисню Give the body an elevated position, oxygen inhalation

Введення бронхолітиків Introduction of bronchodilators

Дихання через маску, інфузія розчину глюкози Breathing through a mask, infusion of glucose solution

ШВЛ, інфузійна терапія Ventilator, infusion therapy

150 / 200
Лікарем швидкої допомоги у лікарню доставлена вагітна 37 тижнів із скаргами на сильний головний біль, утруднене дихання через ніс, 'мерехтінням мушок'перед очима, біль у епігастрії. Вдома були тоніко-клонічні судоми. Об’єктивно: генералізо-вані набряки, АТ- 190/110 мм рт.ст. Про-теїнурія - 5 г/л у разовій порції сечі. Стан плоду задовільний. Яка тактика ведення вагітної? A 37-week pregnant woman was brought to the hospital by an emergency doctor with complaints of a severe headache, difficulty breathing through the nose, 'flickering of flies' in front of the eyes, pain in the epigastrium. At home there were tonic-clonic convulsions. Objectively: generalized edema, blood pressure - 190/110 mm Hg. Proteinuria - 5 g/l in a single portion of urine. The condition of the fetus is satisfactory. What are the tactics of managing a pregnant woman?

Накладання акушерських щипців Applying obstetric forceps

Рання амніотомія Early amniotomy

Терміновий кесарський розтин Urgent cesarean section

Родорозрішення через природні пологові шляхи Dissolution through natural birth canals

Гіпотензивна терапія Hypotensive therapy

151 / 200
Вагітна у 37 тижнів доставлена своїм транспортом із скаргами на сильний го- ловний біль, погіршення зору. Об’єктивно: анасарка, АТ- 200/120 мм рт.ст., білок сечі - 6,6 г/л. На приймальному покої з’явились посмикування м’язів обличчя, тонічні та клонічні судоми, знепритомніла. У свідомість прийшла через 2 хвилини. Серцебиття плоду 100/хв. Яка тактика ведення вагітної? A 37-week pregnant woman was delivered by her own transport with complaints of a severe headache, impaired vision. Objectively: anasarca, blood pressure - 200/120 mm Hg. century, urine protein - 6.6 g/l. Facial muscle twitching, tonic and clonic convulsions appeared in the waiting room, she fainted. She regained consciousness after 2 minutes. Fetal heartbeat 100/min. What are the tactics of managing a pregnant woman?

Ургентний кесарський розтин Emergency caesarean section

Консервативне ведення пологів Conservative management of childbirth

Інтенсивна терапія протягом 24 годин Intensive therapy for 24 hours

Інтенсивна терапія протягом 1 години Intensive therapy for 1 hour

Інтенсивна терапія протягом 2 годин Intensive therapy for 2 hours

152 / 200
У хлопчика 3-х років після перенесеного гострого ентероколіту ешеріхіозної етіології виявляються іктеричність склер та шкіри, набряки під очима, на нижніх кінцівках, гепатоспленомегалія. У крові: ер.-2, 8 • 1012/л, Hb- 82 г/л, тромб.- 140 • 109/л, непрямий білірубін - 78 мкмоль/л, прямий білірубін - 5 мкмоль/л, креатинін - 170 мкмоль/л. Сеча темна, добовий діурез 150 мл, ер.-10-12 в п/з, білок - 0,7 г/л. Про який діагноз слід думати? In a 3-year-old boy, after acute enterocolitis of escherichia etiology, icterus of the sclera and skin, edema under the eyes, on the lower extremities, hepatosplenomegaly are detected. In the blood: er. -2, 8 • 1012/l, Hb- 82 g/l, thromb.- 140 • 109/l, indirect bilirubin - 78 μmol/l, direct bilirubin - 5 μmol/l, creatinine - 170 μmol/l. Urine is dark , daily diuresis 150 ml, er.-10-12 in p/z, protein - 0.7 g/l. What diagnosis should we think about?

Гемолітико-уремічний синдром Hemolytic uremic syndrome

Гострий вірусний гепатит Acute viral hepatitis

Синдром Рея Ray syndrome

Гострий пієлонефрит Acute pyelonephritis

Гострий гломерулонефрит Acute glomerulonephritis

153 / 200
До приймального відділення доставлена група дітей з вираженими явищами ексикозу ( загальна слабкість, часті рідкі випорожнення та блювання). Захворіли гостро. Випорожнення водянисті, рясні, 15-20 разів, з плаваючими пластівцями, мають вигляд 'рисового відвару'. Діти доставлені з вогнища епідемічного спалаху кишкової інфекції ( холера?). Який метод лабораторного обстеження є найбільш інформативним? A group of children with pronounced symptoms of exicosis (general weakness, frequent liquid stools and vomiting) were brought to the reception department. They became acutely ill. The stools are watery, abundant, 15-20 times , with floating flakes, have the appearance of 'rice broth'. Children were delivered from the epicenter of an epidemic outbreak of intestinal infection (cholera?). Which method of laboratory examination is the most informative?

Бактеріологічний метод Bacteriological method

Діагностика методом ультразвукового дослідження Ultrasonic diagnosis

Серологічний метод виявлення зростання титру специфічних антитіл Serological method of detecting the increase in the titer of specific antibodies

Мікроскопія тонкого мазку та товстої краплі крові Microscopy of a thin smear and a thick drop of blood

Біохімічний метод Biochemical method

154 / 200
Вагітна в терміні 33 тижні звернулася до лікаря із скаргами на відсутність рухів у плода. Об’єктивно: матка збільшена до 32-33 тижнів вагітності, в нормотонусі, пе-редлежить голівка, серцебиття плоду не вислуховується. Яке ускладнення виникло? A 33-week pregnant woman turned to the doctor with complaints about the lack of movement in the fetus. Objectively: the uterus was enlarged to 32-33 weeks of pregnancy, normotonic, pe - the head is lying down, the heartbeat of the fetus cannot be heard. What complication occurred?

Передчасні пологи Premature birth

Антенатальна загибель плоду Antenatal death of the fetus

Синдром затримки розвитку плоду Fetal retardation syndrome

Інтранатальна загибель плоду Intranatal death of the fetus

Вроджена вада розвитку плоду Congenital fetal malformation

155 / 200
Хворий 52-х років скаржиться на ін- тенсивний стискаючий біль в ділянці серця, загальну слабість, запаморочення, нудоту, задишку в спокої. Об’єктивно: загальний стан важкий, шкірні покриви бліді, акроці-аноз. АТ- 70/20 мм рт.ст., Ps- 110/хв. Який препарат слід використати в першу чергу для корекції артеріального тиску? A 52-year-old patient complains of intense squeezing pain in the region of the heart, general weakness, dizziness, nausea, shortness of breath at rest. Objectively: general condition severe, pale skin, acrocy-anosis. Blood pressure - 70/20 mm Hg, Ps - 110/min. What drug should be used first of all to correct blood pressure?

Кофеїн Caffeine

Сульфокамфокаїн Sulfocamphocaine

Строфантин Strophantin

Допамін Dopamine

Адреналін Adrenaline

156 / 200
Бригадою швидкої допомоги доставлено підлітка, який скаржиться на різкий, кинджальний біль у лівій половині калитки. Захворів раптово, годину тому під час гри в футбол. Об’єктивно: калитка візуально не змінена, різка болючість при спробі пальпації лівого яєчка. По даних УЗД яєчка та придатки в межах норми. У крові та сечі: в межах норми. Яка патологія в даного хворого? A teenager was brought in by ambulance, complaining of a sharp, stabbing pain in the left half of the wicket. He fell ill suddenly, an hour ago, while playing football. Objectively: the sphincter is visually unchanged, sharp pain when palpating the left testicle. According to ultrasound, the testicles and appendages are within normal limits. In blood and urine: within normal limits. What pathology does this patient have?

Фунікуліт Funiculitis

Перекрут лівого яєчка Left testicular torsion

Варикоцеле Varicocele

Орхіт Orchitus

Защемлена пахова кила Squeezed inguinal hernia

157 / 200
Жінка 22-х років викликала бригаду швидкої допомоги. Скаржиться на значні кров’янисті виділення з піхви зі згортками, переймоподібні болі в низу живота. Об’єктивно: АТ110/60 мм рт.ст., Ps- 90/хв. Остання менструація 2 місяці тому. Біма-нуальне обстеження : канал шийки матки вільно пропускає палець. Матка збільшена до 7 тижнів вагітності, болюча. Додатки не змінені, склепіння вільні, виділення кров’янисті, помірні. Який діагноз найбільш імовірний? A 22-year-old woman called an ambulance. She complains of significant vaginal bleeding with clots, spasm-like pain in the lower abdomen. Objectively: AT110 /60 mm Hg, Ps- 90/min. Last menstruation 2 months ago. Bi-nual examination: the cervical canal freely passes a finger. The uterus is enlarged until 7 weeks of pregnancy, painful. The appendages are not changed, the vaults are free, blood discharge 'bright, moderate. What is the most likely diagnosis?

Апоплексія яєчника Ovarian apoplexy

Сальпінгоофорит Salpingoophoritis

Позаматкова вагітність Ectopic pregnancy

Неповний аборт Incomplete abortion

Дисфункціональна маткова кровотеча Dysfunctional uterine bleeding

158 / 200
У вагітної 36-ти років раптово виникли біль в грудній клітці, почуття страху, ядуха, кашель, кровохаркання. До цього непокоїли слабкість, субфебрилітет, біль в низу живота, що посилюється під час ходи. Об’єктивно: збуджена, бліда, вени шиї пульсують. Попередній діагноз: тромбоем-болія легеневої артерії. Яка першочергова діагностична тактика? A 36-year-old pregnant woman suddenly developed chest pain, a feeling of fear, nausea, coughing, hemoptysis. Before this, weakness, low-grade fever, pain in the lower abdomen were disturbing , which increases during walking. Objectively: excited, pale, neck veins pulsate. Previous diagnosis: thromboembolism of the pulmonary artery. What is the primary diagnostic tactic?

Ангіопульмонографія, рентгенограма грудної клітки із захистом ділянки живота Angiopulmonography, X-ray of the chest with protection of the abdomen

Ангіопульмонографія, ЕКГ Angiopulmonography, ECG

ЕКГ ECG

Рентгенограма грудної клітки із захистом ділянки живота Rentogram of the chest with protection of the abdomen

ЕКГ, рентгенограма грудної клітки із захистом ділянки живота ECG, chest x-ray with protection of the abdomen

159 / 200
У пацієнта 50-ти років з цирозом печінки у стадії декомпенсації після вживання алкоголю розвинулась жовтяниця. Через тиждень було відмічено запаморочення, ляскаючий тремор, солодкий запах у видихуваному повітрі. Діагноз: гостра печінкова недостатність. У яке відділення слід госпіталізувати хворого? A 50-year-old patient with liver cirrhosis in the decompensation stage developed jaundice after drinking alcohol. A week later, dizziness, clapping tremors, and a sweet smell in exhaled air were noted. Diagnosis: acute liver failure. In which department should the patient be hospitalized?

Хірургічне Surgical

Інфекційне Infectious

Реанімаційне Resuscitation

Гастроентерологічне Gastroenterology

Терапевтичне Therapeutic

160 / 200
Дівчинка 2-х років госпіталізована в реанімаційне відділення з приводу стенозу-ючого ларинготрахеобронхіту, стенозу ІІІ ступеня. Об’єктивно: свідомість сплутана. Інспіраторна задишка. В акті дихання бере участь допоміжна мускулатура. У легенях дихання не прослуховується. Тони серця приглушені, тахікардія. Призначення преднізолону не дало позитивного ефекту. Яка подальша лікувальна тактика? A 2-year-old girl was hospitalized in the intensive care unit due to stenosing laryngotracheobronchitis, III degree stenosis. Objectively: consciousness is confused. Inspiratory shortness of breath. In the act of breathing accessory musculature is involved. Breathing is not heard in the lungs. Heart sounds are muffled, tachycardia. The appointment of prednisolone did not have a positive effect. What is the further treatment strategy?

Інтубація трахеї Tracheal intubation

Призначення клемастіну Prescription of Clemastine

Повторне призначення преднізолону Prednisone Prescription

Призначення еуфіліну Appointment of euphilin

Призначення но-шпи No-Spy Assignment

161 / 200
Хвора 45-ти років страждає на тиреотоксикоз, антитиреоїдну терапію приймає нерегулярно. Після перенесеного ГР-ВІ скаржиться на серцебиття, пітливість, повторне блювання, пронос, різку слабість, млявість. Об’єктивно: температура тіла 38,9°C, Ps140 /хв., слабкого наповнення. Тони серця приглушені, миготлива аритмія. У легенях поодинокі сухі хрипи. Який стан розвинувся у хворої? A 45-year-old patient suffers from thyrotoxicosis, takes antithyroid therapy irregularly. After suffering acute respiratory viral infection, she complains of palpitations, sweating, repeated vomiting, diarrhea, severe weakness, lethargy. Objectively: body temperature 38.9°C, Ps140/min., weak filling. Heart sounds are muffled, pulsating arrhythmia. Single dry rales in the lungs. What condition has the patient developed?

Запалення легень Pneumonia

Харчове отруєння Food poisoning

Тиреотоксична криза Thyrotoxic crisis

Міокардит Myocarditis

Передозування антитиреоїдних препаратів Overdose of antithyroid drugs

162 / 200
Під час відпочинку на річці, молодий чоловік здійснив стрибок з рибальського містка. Скарги на біль, неможливість рухів в шиї. Яка потрібна фіксація при транспортуванні до лікувального закладу? While resting on the river, a young man jumped from a fishing bridge. Complaints of pain, impossibility of movement in the neck. What fixation is required during transportation to a medical facility?

Кільця Дельбе Delbe rings

На сидячих ношах On a sitting stretcher

Шина Крамера Kramer Tire

Руками, підтримуючи за шию Hands supporting the neck

Комір Шанца Schantz Collar

163 / 200
18- ти річна дівчина звернулася до лікаря-гінеколога з приводу зґвалтування, яке сталося 2 години потому, коли вона поверталась з університету. Куди треба направити дівчину для підтвердження факту зґвалтування? An 18-year-old girl consulted a gynecologist about a rape that happened 2 hours later when she was returning from university. Where should the girl be referred to confirm the fact rape?

В жіночу консультацію за місцєм проживання To a women's consultation at the place of residence

На судово-медичну експертизу For forensic examination

До психiатра To the psychiatrist

До місцевого РУ ГУ МВС To the local RU GU MIA

До сімейного лікаря To the family doctor

164 / 200
Постраждалого доставили до лікарні попутною машиною через 15 хвилин після автокатастрофи. Був прижатий у кабіні перевернутого автомобіля, при цьому отримав опіки кистей рук 'акумуля-торною'кислотою, що вилилася. Перша допомога полягала в обтиранні уражених ділянок снігом та накладанні сухої пов’язки. Скаржиться на сильний біль в місцях ураження. Що треба здійснити в приймальному відділенні лікарні в першу чергу? The victim was taken to the hospital by a passing car 15 minutes after the car accident. He was pinned in the cab of the overturned car, while his hands were burned by 'battery acid', which spilled. The first aid consisted in wiping the affected areas with snow and applying a dry bandage. He complains of severe pain in the affected areas. What should be done in the hospital admissions department first of all?

Виконати туалет опікових ділянок розчином перекису водню Perform toileting of burn areas with hydrogen peroxide solution

Провести обливання кистей рук у проточній воді протягом 30-40 хвилин Rinse hands in running water for 30-40 minutes

Знеболювання Analgesia

Накласти пов’язку з водорозчинною маззю Apply a bandage with a water-soluble ointment

Накласти напівспиртову пов’язку Apply semi-alcohol bandage

165 / 200
Під час обіду в кафе групи студентів, один з них раптово припинив розмовляти, схопився за шию, виник ціаноз. Який діагноз і заходи першої допомоги? During lunch in a cafe of a group of students, one of them suddenly stopped talking, grabbed his neck, cyanosis occurred. What is the diagnosis and first aid measures?

Набряк легень. Введення сечогінних Pulmonary edema. Administration of diuretics

Ларингоспазм. Введення спазмолітиків Laryngospasm. Administration of antispasmodics

Гострий коронарний синдром. Прийом валідолу, нітрогліцерину per os Acute coronary syndrome. Taking validol, nitroglycerin per os

Аспірація стороннього тіла. Проведення прийому Геймліха Foreign body aspiration. Heimlich administration

Приступ бронхіальної астми. Інгаляція сальбутамолу Bronchial asthma attack. Salbutamol inhalation

166 / 200
Чоловік 40-ка років, скаржиться на багаторазове блювання, що з’явилось через 30 хвилин після опромінення на АЕС, біль у животі, озноб, високу температуру -38 — 39oC, зниження артеріального тиску. На 2-й день з’явився пронос. Доза загального опромінення 20 Гр. Яка форма гострої променевої хвороби розвинулась? A 40-year-old man complains of repeated vomiting that appeared 30 minutes after exposure to the nuclear power plant, abdominal pain, chills, high temperature -38 — 39oC, decrease in blood pressure. Diarrhea appeared on the 2nd day. The total radiation dose was 20 Gy. What form of acute radiation sickness developed?

Церебральна форма Cerebral form

Кишково-мозкова форма Intestinal form

Це прояви не променевої хвороби This is not a manifestation of radiation sickness

Токсемічна форма Toxemic form

Кишкова форма Intestinal form

167 / 200
На хімічно-небезпечному об’єкті сталася аварія з викидом сильнодіючих отруйних речовин (СДОР). Персонал медичного пункту хімічного підприємства та бригада ШМД, які прибули до вогнища масових санітарних втрат, розпочали надання першої медичної допомоги потерпілим від впливу СДОР Які заходи НЕ ВХОДЯТЬ до обсягу першої медичної допомоги? At a chemically hazardous facility, an accident occurred with the release of highly toxic substances (PSS). The personnel of the medical center of the chemical enterprise and the team of the SMD, who arrived at the outbreak of mass sanitary losses, started providing first medical aid to victims of the effects of SDR. What measures are NOT included in the scope of first medical aid?

Оксигенотерапія Oxygenotherapy

Тимчасова зупинка зовнішньої кровотечі Temporary stoppage of external bleeding

Накладання оклюзійної пов’язки при відкритому пневмотораксі Applying an occlusive dressing for open pneumothorax

Одягання протигазу на ураженого, який перебуває на зараженій території Dressing a gas mask on a victim who is in an infected area

Введення знеболюючих засобів Injection of painkillers

168 / 200
У пацієнта відзначається спазм кистей і стоп, болісні тонічні судоми, аж до судомних припадків, порушення дихання, позитивні симптоми Хвостека і Трусо. Спостерігаються профузне потовиділення, брон-хоспазм, пронос, поліурія, стан гострого психічного розладу, сухість і лущення шкіри, множинний карієс зубів, катаракта. На ЕКГ - суттєве подовження електричної систоли серця (інтервал QT) і сплощення зубця Т. Рівень кальцію у сироватці крові -1,55 ммоль/л. Який попередній діагноз? The patient has a spasm of the hands and feet, painful tonic spasms, up to convulsive seizures, breathing disorders, positive symptoms of Khvostek and Trousseau. Profuse sweating, bronchospasm are observed , diarrhea, polyuria, a state of acute mental disorder, dryness and peeling of the skin, multiple caries of the teeth, cataracts. On the ECG, there is a significant prolongation of the electrical heart systole (QT interval) and flattening of the T wave. The level of calcium in the blood serum is -1.55 mmol/l . What is the previous diagnosis?

Синдром мальабсорбції Malabsorption syndrome

Гіперпаратиреоз Hyperparathyroidism

Нормокальціємічна тетанія Normocalcemic tetany

Ниркова недостатність Kidney failure

Гіпотиреоз Hypothyroidism

169 / 200
До відділення реанімації доставлено чоловіка 55-ти років, який хворіє на цукровий діабет понад 10 років. Об’єктивно: непритомний, шкіра та слизові оболонки сухі, дихання поверхневе, часте, арефлексія. ЧСС- 132/хв., АТ- 80/40 мм рт.ст., Hb- 160 г/л, лейк.- 10,2 • 109 /л, Ht- 0,64, цукор крові - 45,5 ммоль/л, K- 6,2 ммоль/л, Na- 162 ммоль/л. Ацетон в сечі не виявлений. Тиждень тому в зв’язку з загостренням панкреатиту та вираженим болем у животі зменшив кількість їжі та удвічі зменшив дозу інсуліну. Який аналіз необхідно зробити, щоб уточнити діагноз? A 55-year-old man with diabetes for more than 10 years was brought to the intensive care unit. Objectively: unconscious, dry skin and mucous membranes, shallow breathing , frequent, areflexia. Heart rate - 132/min., BP - 80/40 mm Hg, Hb - 160 g/l, leuk. - 10.2 • 109 /l, Ht - 0.64, blood sugar - 45.5 mmol/l, K- 6.2 mmol/l, Na- 162 mmol/l. Acetone was not detected in the urine. A week ago, due to exacerbation of pancreatitis and severe abdominal pain, he reduced the amount of food and halved the dose insulin. What analysis should be done to clarify the diagnosis?

Креатинін, сечовина крові Creatinine, blood urea

рН крові blood pH

Визначення осмолярності плазми Determination of plasma osmolarity

Аналіз сечі на добову глюкозурію Urine analysis for daily glucosuria

Аналіз сечі на добову протеїнурію Urine analysis for daily proteinuria

170 / 200
Пацієнтка 82-х років знайдена після землетрусу серед зруйнованої будівлі. Тривалість перебування в завалі - 8 годин, температура навколишнього середовища +5oC. Діагностовано порушення серцевого ритму - брадикардія з подальшою зупинкою серцевої діяльності. Які заходи треба вжити негайно? An 82-year-old patient was found after an earthquake in the midst of a destroyed building. The duration of her stay in the rubble was 8 hours, the ambient temperature was +5oC. A heart rhythm disorder was diagnosed - bradycardia with further cardiac arrest. What measures should be taken immediately?

Забезпечення прохідності дихальних шляхів, штучна вентиляція легень, непрямий масаж серця, внутрішньовенно атропін 0,1% - 3 мл Ensuring airway patency, artificial lung ventilation, indirect heart massage, intravenous atropine 0.1% - 3 ml

Дефібриляція Defibrillation

Ввести теплу плазму внутрішньовенно Inject warm plasma intravenously

Великі дози глюкокортикоїдів внутрішньовенно Large doses of intravenous glucocorticoids

Лідокаїн 1 мг/кг внутрішньовенно Lidocaine 1 mg/kg IV

171 / 200
Під час забезпечення центрального ве- нозного доступу при транспортуванні постраждалого з місця катастрофи відмічено розвиток підшкірної ємфізєми ділянки шиї та грудної клітки. Яке обстеження необхідно провести постраждалому в першу чергу в приймальному відділенні лікарні? During the provision of central venous access during transportation of the victim from the accident site, the development of subcutaneous emphysema of the neck and chest area was noted. What examination should be performed on the victim first of all in reception department of the hospital?

Електроенцефалографія Electroencephalography

Газовий склад крові Blood gas composition

Ультразвукове дослідження черевної порожнини Ultrasound examination of the abdominal cavity

Рентгенографія органів грудної порожнини Rentgenography of chest cavity organs

Електрокардіографія Electrocardiography

172 / 200
Хвора 40-ка років після падіння з висоти другого поверху отримала закриту ЧМТ. Свідомість - помірне приглушення. В зоні потилиці діагностована забійна рана. АТ-110/70 мм рт.ст., ЧСС- 65/хв. Дихання спонтанне, 18/хв. Які особливості транспортування хворої до стаціонару? A patient in her 40s received a closed TBI after falling from the height of the second floor. Consciousness is moderately impaired. In the back of the head, a fatal wound was diagnosed. AT-110/70 mm Hg, heart rate - 65/min. Spontaneous breathing, 18/min. What are the features of transporting the patient to the hospital?

Транспортування в положенні на животі з повернутою вбік головою Transportation in the prone position with the head turned to the side

Транспортування в положенні напівсидя-чи Transportation in semi-sitting position

Транспортування в положенні на спині з накладеним коміром Шанца Transportation in the supine position with a Shantz collar applied

Можливе амбулаторне лікування, госпіталізації не потребує Ambulatory treatment is possible, hospitalization is not required

Транспортування в стабільному боковому положенні Transportation in a stable lateral position

173 / 200
На АЕС відбулася радіаційна аварія з руйнуванням реактора й енергоблоку. У навколишнє середовище було викинуто близько 30 млн. кюрі радіоактивності. При аварії утворилася радіоактивна хмара, у результаті поширення якої відбулося радіоактивне забруднення території, джерел води, продуктів харчування, рослин. Які зони забруднення утворюються на слі-ді радіоактивної хмари? There was a radiation accident at the NPP with the destruction of the reactor and power unit. About 30 million curies of radioactivity were released into the environment. The accident created a radioactive cloud, which spread there was radioactive contamination of the territory, water sources, food products, plants. What zones of contamination are formed in the wake of the radioactive cloud?

Брудна зона, забруднена зона, незабруднена зона, чиста зона Dirty zone, contaminated zone, uncontaminated zone, clean zone

Дуже сильного забруднення, сильного забруднення, середнього забруднення, слабкого забруднення, незабруднена зона Very heavily polluted, heavily polluted, medium polluted, mildly polluted, unpolluted zone

Надзвичайно небезпечного забруднення, небезпечного забруднення, сильного забруднення, помірного забруднення, радіаційної небезпеки Extremely dangerous pollution, dangerous pollution, severe pollution, moderate pollution, radiation danger

Небезпечного забруднення, сильного забруднення, помірного забруднення, безпечного забруднення Dangerous pollution, severe pollution, moderate pollution, safe pollution

Надзвичайного забруднення, помірного забруднення, слабкого забруднення Extreme pollution, moderate pollution, weak pollution

174 / 200
Під час проведення регламентних робіт на АЕС трапилась аварія з викидом радіоактивних речовин. Які заходи колективного захисту у фазі рятування слід застосувати для населення, що перебуває на радіаційно забрудненій території? During routine work at the nuclear power plant, an accident occurred with the release of radioactive substances. What measures of collective protection in the rescue phase should be applied to the population living in a radioactively contaminated area?

Укриття в захисних спорудах Shelter in protective structures

Скоротити час перебування на відкритій місцевості Reduce time spent in open terrain

Розосередження Distribution

В дану фазу колективні засоби захисту можна не застосовувати In this phase collective protection means may not be applied

Евакуація населення з місць ураження Evacuation of the population from affected areas

175 / 200
Під час проведення регламентних робіт на АЕС трапилась аварія з викидом радіоактивних речовин. 32 працівника чергової зміни отримали різні дози зовнішнього опромінення. Які засоби треба використати працівникам з аптечки індивідуальної №2 з радіопротекторною метою? During routine work at the NPP, an accident occurred with the release of radioactive substances. 32 workers on the regular shift received various doses of external radiation. What tools should employees use from the first-aid kit of individual no. 2 with a radioprotective purpose?

Тетрациклін, етаперазин Tetracycline, etaperazin

Цистамін, таблетки йодистого калію Cystamine, potassium iodide tablets

Шприц-тюбик з протибольовим засобом, сульфадиметоксин, тарен Syringe tube with pain reliever, sulfadimethoxine, taren

Шприць-тюбик з протибольовим засобом, тетрациклін Syringe-tube with pain reliever, tetracycline

Тарен, таблетки йодистого калію Taren, potassium iodide tablets

176 / 200
Швидкою допомогою доставлена хвора 67-ми років з діагнозом: закритий перелом нижньої третини правої стегнової кістки із зміщенням відламків. Без транспортної іммобілізації. Загроза виникнення якого ускладнення найбільш імовірна у даної хворої? A 67-year-old patient was delivered by ambulance with a diagnosis: closed fracture of the lower third of the right femur with displacement of fragments. Without transport immobilization. The risk of which complication is most likely in this patient?

Компартмент-синдром Compartment syndrome

Гострий тромбоз судин нижньої кінцівки Acute thrombosis of lower extremity vessels

Пошкодження судинно-нервового пучка Damage of vascular-nerve bundle

Перфорація шкіри Skin Perforation

Незрощення перелому Nonunion of fracture

177 / 200
Захворювання почалося з появи рясних водянистих випорожнень без запаху 9 разів за добу, спраги. Об’єктивно: температура 36,2oC, язик сухий, живіт втягнутий, безболісний. Втратив 6% маси тіла. Запідозрена холера. При яких максимальних втратах можливе використання методу оральної регідратації? The disease began with the appearance of abundant watery stools without odor 9 times a day, thirst. Objectively: the temperature is 36.2oC, the tongue is dry, the stomach is retracted, painless. Lost 6% of body weight. Cholera is suspected. At what maximum losses is it possible to use the oral rehydration method?

7-9% маси тіла 7-9% of body weight

Не можлива Not possible

До 3% маси тіла Up to 3% of body weight

Більше 10% маси тіла More than 10% of body weight

Не більш 7% маси тіла No more than 7% of body weight

178 / 200
Раптово на вулиці у жінки 60-ти років наступила втрата свідомості та апное. Пульсація сонних артерій визначається. При спробі проведення ШВЛ методом 'рот до рота'виявлене утруднення вдиху. Шкірні покриви хворої залишаються ціаноти-чними. Які заходи необхідно провести для підвищення ефективності реанімаційної допомоги? Suddenly, a 60-year-old woman lost consciousness and apnoea on the street. The pulsation of the carotid arteries is determined. During an attempt to perform mouth-to-mouth ventilation, difficulty in breathing was detected The patient's skin remains cyanotic. What measures should be taken to increase the effectiveness of resuscitation care?

Збільшити частоту вдмухувань Increase the frequency of inhalations

Натискати на м’язи черевного преса Press the abdominal muscles

Натискати на верхню третину груднини Press on the upper third of the sternum

Відновити прохідність дихальних шляхів Restore airway patency

Зменшити частоту вдмухувань Reduce the frequency of inhalations

179 / 200
Жінка 80-ти років під час сварки з родичами раптово зблідла, схопилася за ділянку серця, через 5 хвилин впала непритомна. Оглянута лікарем швидкої допомоги через 25 хвилин пульс і тиск не визначаються. Відомо, що впродовж останніх 20-ти років лікувалася з приводу стенокардії. Які мають бути дії бригади швидкої допомоги? During an argument with her relatives, a woman in her 80s suddenly turned pale, grabbed her heart area, fell unconscious after 5 minutes. She was examined by an emergency doctor 25 minutes later, her pulse and pressure is not determined. It is known that during the last 20 years she was treated for angina pectoris. What should be the actions of the ambulance team?

Зробити закритий масаж серця Make a closed heart massage

Провести комплекс реанімаційних заходів Conduct a set of resuscitation measures

Зробити ЕКГ Make an ECG

Не проводити ніяких заходів і зафіксувати смерть Do not take any action and record death

Терміново госпіталізувати в кардіологічне відділення Urgent hospitalization in the cardiology department

180 / 200
В клініку доставлена хвора 82-х років зі скаргами на серцебиття, задишку, загальну слабість. Стан раптово погіршився за кілька годин до госпіталізації, коли з’явилося серцебиття, кардіалгії, пізніше задишка. Раніше проводилось лікування з приводу хронічної серцевої недостатності. Об’єктивно: акроціаноз, АТ- 130/80 мм рт.ст. Діяльність серця аритмічна, тони серця глухі, систолічний шум на верхівці, ЧСС- 126/хв. В нижніх відділах легень дрі-бнопухирчасті вологі хрипи. На ЕКГ - фібриляція передсердь з тахісистолією шлуночків. Які оптимальні добові дози інфузій дигоксину слід призначити? An 82-year-old patient was brought to the clinic with complaints of palpitations, shortness of breath, general weakness. The condition suddenly worsened a few hours before hospitalization, when palpitations appeared, cardiac pain, later shortness of breath. Previously treated for chronic heart failure. Objectively: acrocyanosis, blood pressure - 130/80 mm Hg. Heart activity is arrhythmic, heart sounds are dull, systolic murmur at the apex, heart rate - 126/min. B in the lower parts of the lungs, small-vesicular wet rales. On the ECG - atrial fibrillation with ventricular tachysystole. What optimal daily doses of digoxin infusions should be prescribed?

По 2 мл в/в 0,025% розчину раз на добу 2 ml IV 0.025% solution once a day

По 0,5 мл 2 рази на добу 0,025% розчину в/в 0.5 ml 2 times a day of 0.025% IV solution

По 0,5 мл 0,025% в/в раз на добу 0.5 ml of 0.025% IV once a day

По 1 мл 0,025% в/в двічі на добу до 3-4-х днів, поступово знижуючи дозу 1 ml of 0.025% IV twice a day for 3-4 days, gradually reducing the dose

По 1 мл 0,025% розчину в/в раз на добу 1 ml of 0.025% IV solution once a day

181 / 200
На занятті з фізкультури учень отримав травму грудної клітки, виражена болючість в ділянці 6,7, 8 ребер зліва. Як слід переносити і транспортувати хворого? During a physical education class, a student suffered a chest injury, severe pain in the area of ​​6, 7, 8 ribs on the left. How should the patient be carried and transported?

Лежачи на лівому боці Lying on the left side

Лежачи на животі Lying on your stomach

Лежачи на правому боці Lying on right side

В напівсидячому положенні In a semi-sitting position

Лежачи на спині Lying on your back

182 / 200
Хлопчик 12-ти років під час гри у футбол при падінні отримав травму. Встановлено попередній діагноз: закритий вивих правого передпліччя. Який об’єм допомоги повинен надати лікар на місці пригоди? A 12-year-old boy was injured during a fall while playing football. A preliminary diagnosis was made: closed dislocation of the right forearm. What amount of care should the doctor provide for place of adventure?

Восьмиподібна пов’язка на правий ліктьовий суглоб Eight-shaped bandage on the right elbow joint

Косинкова пов’язка на праву верхню кінцівку Handkerchief bandage on the right upper limb

Знеболювання, транспортна іммобілізація косинкою Anesthesia, transport immobilization with a scarf

Знеболювання, спроба вправлення вивиху Anesthesia, attempt to repair dislocation

Знеболювання, транспортна іммобілізація шиною Anesthesia, transport immobilization with a splint

183 / 200
Жінка у терміні вагітності 38 тижнів доставлена до пологового відділення із скаргами на відсутність рухів плоду протягом трьох днів та переймоподібні болі в низу живота, що розпочались 6 годин тому назад. Розміри тазу 26-28-31-21 см., серцебиття плоду не визначається. Передлежан-ня головне. Активна пологова діяльність, розкриття шийки матки майже повне. Яка тактика лікаря? A 38-week pregnant woman was brought to the maternity ward with complaints of no fetal movement for three days and cramp-like pains in the lower abdomen that started 6 hours ago. The dimensions of the pelvis are 26-28-31-21 cm, the heartbeat of the fetus is not determined. Presentation is the main thing. Active labor, the opening of the cervix is ​​almost complete. What are the doctor's tactics?

Плодоруйнівна операція (краніотомія) Fetal destruction operation (craniotomy)

Вакуум-екстракція плоду Vacuum fruit extraction

Кесарів розтин Caesarean section

Плодоруйнівна операція (декапітація) Fruit destruction operation (decapitation)

Очікувальна тактика Waiting tactics

184 / 200
У новонародженої дитини констатовано гемолітичну анемію. В венозній крові: Ht- 28%, Hb- 95 г/л. Для корекції анемії слід обрати: A newborn child was diagnosed with hemolytic anemia. In venous blood: Ht- 28%, Hb- 95 g/l. To correct anemia, you should choose:

Актиферин Aktiferin

Цільна кров Whole blood

Фероплекс Feroplex

Еритроцитарна маса Erythrocyte mass

Гемофер Hemofer

185 / 200
У хворої 67-ми років з цукровим діабетом 2 типу опіки І-ІІ ст. (25% поверхні тіла). В стаціонарі розвився ступор, шкіра неушкоджених ділянок суха, зморшкувата, тургор знижений, інколи виникають судоми. Температура тіла - 37,3oC, Ps- 114/хв., АТ- 85/45 мм рт.ст. Живіт м’який. У крові: цукор - 54 ммоль/л, рН- 7,35, осмолярність -355 мосмоль/л. В чому полягає першочергова невідкладна допомога? A 67-year-old patient with diabetes type 2 has burns of the 1st-2nd degree (25% of the body surface). Stupor has developed in the hospital, the skin of the intact areas is dry , wrinkled, turgor is reduced, sometimes convulsions occur. Body temperature - 37.3oC, Ps - 114/min., BP - 85/45 mm Hg. Abdomen is soft. In blood: sugar - 54 mmol/l, pH - 7.35, osmolarity -355 mosmol/l. What is the primary emergency care?

Введення нео-гемодезу і 60 ОД інсуліну Introduction of neo-hemodesis and 60 units of insulin

Введення гіпотонічного 2,5% розчину глюкози і 100 мг гідрокортизону Introduction of hypotonic 2.5% glucose solution and 100 mg of hydrocortisone

Введення 5% розчину глюкози і 60 мг лазиксу Introduction of 5% glucose solution and 60 mg of Lasix

Введення ізотонічного 0,9% розчину NaCl і 0,5 мл 0,05% розчину строфантину Introduction of isotonic 0.9% NaCl solution and 0.5 ml of 0.05% strophanthin

Введення гіпотонічного 0,45% розчину NaCl і інсуліну за 'схемою малих доз' Introduction of hypotonic 0.45% NaCl solution and insulin according to the 'scheme of small doses'

186 / 200
Чоловік 28-ми років потрапив у ДТП. Під час огляду лікарем ШМД постражда-лий скаржиться на нудоту, запаморочення, інтенсивний біль у нижніх кінцівках. Об’єктивно: АТ- 75/50 мм рт.ст., ЧСС- 130/хв, ЧД- 26/хв. Діагностовано політравму. Чим обумовлений стан хворого? A 28-year-old man was involved in a road accident. During the examination by a doctor at the Medical Center, the victim complained of nausea, dizziness, and intense pain in the lower extremities. Objectively: Blood pressure - 75/50 mm Hg, heart rate - 130/min, HR - 26/min. Polytrauma was diagnosed. What is the cause of the patient's condition?

Гостра кровотеча, розвиток жирової емболії, гострий больовий синдром Acute bleeding, development of fat embolism, acute pain syndrome

Гостра дихальна недостатнїсть Acute respiratory failure

Гостра надниркова недостатність Acute adrenal insufficiency

Черепно-мозкова травма Traumatic brain injury

Гостра серцева недостатнїсть Acute heart failure

187 / 200
У хворого 20-ти років вчора виник біль у промежині, який іррадіював у пряму кишку. За останні 16 годин самостійне сечовипускання відсутнє. Дві доби тому знаходився на зимовій рибалці. Яка тактика надання першої медичної допомоги? A 20-year-old patient had pain in the perineum yesterday, which radiated to the rectum. There has been no spontaneous urination for the past 16 hours. Two days ago he was on a winter fishing trip What are the tactics of providing first aid?

Катетеризація сечового міхура катетером Нейлятона Catheterization of the urinary bladder with a Neilaton catheter

Троакарна цистостомія, госпіталізація в урологічне відділення Trocar cystostomy, hospitalization in the urology department

Виділення сечі з встановленням катетера Фолея Urine excretion with Foley catheter

Призначення антибактеріальних препаратів групи фторхінолонів Prescription of antibacterial drugs of the fluoroquinolone group

Призначення антибактеріальних препаратів групи макролідів Prescription of antibacterial drugs of the macrolide group

188 / 200
В реанімаційному відділенні стан хворого зненацька погіршився: свідомість відсутня, зупинка дихання. На ЕКГ - ізолінія. Який з перелічених нижче заходів ПРОТИПОКАЗАНИЙ у даному випадку? In the intensive care unit, the patient's condition suddenly worsened: there is no consciousness, breathing has stopped. There is an isoline on the ECG. Which of the measures listed below is CONTRAINDICATED in this case?

Непрямий масаж серця Indirect heart massage

Введення атропіну Atropine administration

Введення адреналіну Injection of adrenaline

Електрична дефібриляція Electrical defibrillation

Інтубація трахеї та ШВЛ Tracheal intubation and ventilator

189 / 200
У хворого під час епістатусу виникла зупинка дихання та кровообігу. Бригада ШМД прибула через 25 хвилин. Об’єктивно: дихання відсутнє, пульс на магістральних судинах відсутній, зіниці широкі, шкіра сірого кольору. Який найбільш імовірний діагноз? During the epistatus, the patient stopped breathing and blood circulation. The medical team arrived 25 minutes later. Objectively: there is no breathing, there is no pulse on the main vessels, the pupils are wide , gray skin. What is the most likely diagnosis?

Клінічна смерть Clinical death

Кома Comma

Сопор Sopor

Декортикація Decortication

Біологічна смерть Biological death

190 / 200
До приймального відділення доставлено хворого 42-х років зі скаргами на погане самопочуття, порушення зору, слинотечу, бронхорею, м’язові судоми, профузну пітливість, тенезми. Відомо, що такий стан розвинувся після роботи на полі із використанням хлорофосу. Який препарат потрібно ввести негайно? A 42-year-old patient was brought to the reception department with complaints of poor health, impaired vision, drooling, bronchorrhoea, muscle spasms, profuse sweating, tenesmus. It is known , that such a condition developed after working in the field with the use of chlorophos. Which drug should be administered immediately?

Димедрол Diphenhydramine

Атропін Atropine

Прозерин Prozerin

Адреналін Adrenaline

Еуфілін Euphilin

191 / 200
У хворого 67-ми років після психоемоційного стресу порушилась вимова, з’явилося поперхування при ковтанні, зникла чутливість у лівій руці. Об’єктивно: парез м’якого піднебіння зліва, мовної зв’язки, птоз, енофтальм зліва, гемігіпоалгезія зліва. Який попередній діагноз? A 67-year-old patient, after psycho-emotional stress, had impaired pronunciation, difficulty swallowing, sensitivity in the left hand disappeared. Objectively: paresis of the soft palate on the left, tongue ligaments, ptosis, enophthalmos on the left, hemihypoalgesia on the left. What is the previous diagnosis?

Ішемічний інсульт Ischemic stroke

Інфаркт міокарда Myocardial infarction

Пухлина головного мозку Brain tumor

Паренхіматозний крововилив Parenchymal hemorrhage

Субарахноїдальний крововилив Subarachnoid hemorrhage

192 / 200
При огляді педіатром дівчинка 7-ми років, що страждає на гостру мієлобла-стну лейкемію ( другий рецидив), раптово знепритомніла. Шкіра та слизові блідо-ціанотичні. Екскурсій грудної клітки немає. Пульс на магістральних артеріях не визначається. Яка тактика лікаря? During examination by a pediatrician, a 7-year-old girl suffering from acute myeloblastic leukemia (second relapse) suddenly fainted. The skin and mucous membranes are pale cyanotic. Excursions there is no chest. The pulse on the main arteries is not determined. What are the doctor's tactics?

Спитати у батьків, щодо доцільності проведення серцево-легеневої реанімації Ask parents about the feasibility of cardiopulmonary resuscitation

Не розпочинати серцево-легеневу реанімацію Do not start CPR

Розпочати серцево-легеневу реанімацію Start CPR

Терміново порадитись з головним лікарем Urgently consult with the chief physician

Терміново порадитись з юристом Urgently consult a lawyer

193 / 200
Хворий 42-х років переніс операцію з приводу злоякісної пухлини сечового міхура рік тому. Протягом останніх 3-х місяців відмічається слабкість, зниження інтересу до життя. Больовий синдром відсутній. Впродовж останніх 10 днів скаржиться на виділення згустків крові з сечею. Яка найбільш імовірна причина гематурії? A 42-year-old patient underwent an operation for a malignant tumor of the bladder a year ago. During the last 3 months, weakness and a decrease in interest in life have been noted. There is no pain syndrome . During the last 10 days, he complains of blood clots in his urine. What is the most likely cause of hematuria?

Форнікальна кровотеча Fornical bleeding

Пухлина простати Prostate tumor

Інфаркт нирки Kidney infarction

Сечокам’яна хвороба Urolithiasis

Рецидив пухлини сечового міхура Bladder tumor recurrence

194 / 200
У дівчини 17-ти років на холодному морозному повітрі виникають гіперемовані уртикарні висипання на відкритих частинах тіла та на обличчі. Висипання посилюються при вході у тепле приміщення. Який найбільш імовірний діагноз? A 17-year-old girl develops hyperemic urticarial rashes on exposed parts of the body and face in cold frosty air. The rashes worsen when entering a warm room. What is the most likely diagnosis?

Нейродерміт Neurodermatitis

Холінергічна кропивниця Cholinergic urticaria

Теплова кропивниця Heat Urticaria

Холодова кропивниця Cold Urticaria

Медикаментозна кропивниця Medicated urticaria

195 / 200
Чергового анестезіолога викликано до приймального відділення лікарні до дитини, яка декілька годин тому перенесла клінічну смерть, але була успішно реанімована бригадою ШМД. На момент огляду свідома, стан дитини стабільний, вітальні функції компенсовані. До якого відділення необхідно госпіталізувати малюка? The on-duty anesthesiologist was called to the hospital's admissions department for a child who suffered clinical death a few hours ago, but was successfully resuscitated by the emergency medical team. At the time of examination, the child is conscious, the condition of the child is stable , greeting functions are compensated. To which ward should the baby be hospitalized?

Соматичне відділення Somatic Department

Профіль відділення повинен визначити головний лікар The profile of the department must be determined by the chief physician

Профіль відділення повинен визначити старший черговий лікар The department profile must be determined by the senior doctor on duty

Профіль відділення повинен визначити завідувач відділенням реанімації The department profile must be defined by the head of the intensive care unit

Реанімації та інтенсивної терапії Resuscitation and intensive care

196 / 200
Під час проведення гемотрансфузії у дівчинки 9-ти років із геморагічним шоком виникли біль у грудях, попереку, з’явилась задишка, сеча набула червоного забарвлення. Плазма крові після центрифугування рожева. Оберіть оптимальну лікувальну тактику: During hemotransfusion, a 9-year-old girl with hemorrhagic shock developed pain in the chest, lower back, shortness of breath, urine became red. Blood plasma after centrifugation is pink. Choose the optimal treatment tactics:

Припинити трансфузію, розпочати інфу-зію плазмозамінників Stop transfusion, start infusion of plasma substitutes

Продовжувати трансфузію, розпочати інфузію плазмозамінників Continue transfusion, start infusion of plasma substitutes

Припинити трансфузію, ввести кордіамін Stop transfusion, introduce cordiamine

Припинити трансфузію, ввести еуфілін Stop transfusion, enter euphilin

Припинити трансфузію, ввести фуросе-мід Stop transfusion, introduce furosemide

197 / 200
У хлопчика 11-ти років із геморагічним шоком під час проведення гемотрансфузії виникли біль у грудях, попереку, з’явилася задишка, сеча набула червоного забарвлення. Плазма крові після центрифугування рожева. Який найбільш імовірний діагноз? An 11-year-old boy with hemorrhagic shock developed pain in the chest and lower back during hemotransfusion, shortness of breath appeared, and the urine became red. Blood plasma after centrifugation is pink. What is the most likely diagnosis?

Алергічна реакція Allergic reaction

Негемолітична трансфузійна реакція Nonhemolytic transfusion reaction

Пірогенна реакція Pyrogenic reaction

Синдром масивних трансфузій Massive transfusion syndrome

Гострий гемоліз Acute hemolysis

198 / 200
Хворий 54-х років доставлений в приймальне відділення інфекційної лікарні у тяжкому стані. Дезорієнтований у просторі та часі, багатослівний, неадекватно реагує на оточуючих. Об’єктивно: температура тіла 40,2oC, обличчя набрякле, гіпе-ремоване, виражений кон’юнктивіт та ін’єкція судин склер. На шкірі грудей, спини, бокових поверхонь тулуба та згинальних поверхонь кінцівок рясний розеольозно-петехіальний висип. Ps- 132/хв., слабкого наповнення, аритмічний, АТ- 75/40 мм рт.ст. Дихання часте, поверхневе. Позитивні симптом Говорова-Годельє та менінге-альні симптоми. Лікування необхідно розпочати із: A 54-year-old patient was brought to the reception department of an infectious disease hospital in serious condition. He is disoriented in space and time, verbose, reacts inadequately to others. Objectively: temperature body 40.2oC, the face is swollen, hyperemic, pronounced conjunctivitis and injection of scleral vessels. On the skin of the chest, back, lateral surfaces of the trunk and flexor surfaces of the limbs, abundant roseolous-petechial rash. Ps- 132/min., weak filling, arrhythmic, blood pressure - 75/40 mm Hg. Breathing is frequent, shallow. Positive Hovorov-Godelier symptom and meningeal symptoms. Treatment should be started with:

Внутрішньом’язового введення 2 мл 2,5% р-ну аміназину Intramuscular injection of 2 ml of 2.5% p-nu aminazine

Внутрішньовенного введення 1,0 мл 0,2% р-ну норадреналіну Intravenous administration of 1.0 ml of 0.2% noradrenaline

Внутрішньом’язового введення 1 г левоміцетину сукцинату Intramuscular administration of 1 g of chloramphenicol succinate

Внутрішньовенного введення 400 мл р-ну реосорбілакту Intravenous administration of 400 ml of rheosorbilact

Внутрішньом’язового введення 60 мг преднізолону Intramuscular administration of 60 mg of prednisolone

199 / 200
У хворого 25-ти років субфебрильна температура, біль у животі, рідкі випорожнення з домішками слизу до 3-4 разів на добу. Хворіє протягом 3-х діб. Язик вологий, обкладений білим нальотом. Живіт м’який, болючий в лівій здухвинній ділянці, сигмоподібна кишка спазмована, болюча. Який метод дослідження буде найбільш інформативним для постановки діагнозу? A 25-year-old patient has a low-grade fever, abdominal pain, loose stools with mucus up to 3-4 times a day. He has been ill for 3 days. The tongue is moist, coated with a white coating. The abdomen is soft, painful in the left pubic region, the sigmoid colon is spasmodic, painful. What research method will be the most informative for making a diagnosis?

Загальний аналіз сечі General urinalysis

Копроцитограма Coprocytogram

Копрокультура Coproculture

Загальний аналіз крові General blood test

РНГА RNHA

200 / 200
Хвора повернулась після відпочинку з однієї з країн Близького Сходу, де перебувала впродовж місяця. Захворіла гостро, звернулась до лікаря з приводу нападів, які супроводжувались підвищенням температури тіла, лихоманкою, рясним потовиділенням. Під час огляду шкіра бліда, з жовтяничним відтінком; печінка і селезінка дещо збільшені. Яка тактика сімейного лікаря? The patient returned after a vacation from one of the countries of the Middle East, where she stayed for a month. She became acutely ill, went to the doctor about seizures, which were accompanied by an increase in body temperature, fever , profuse sweating. During the examination, the skin is pale, with a yellowish tinge; the liver and spleen are slightly enlarged. What are the family doctor's tactics?

Негайно госпіталізувати в реанімаційне відділення Immediate hospitalization in the intensive care unit

Амбулаторне лікування Outpatient treatment

Направити до інфекційного стаціонару Send to infectious hospital

Спостереження в умовах ізолятору поліклініки Observation in the isolation ward of the polyclinic

Направити до гематологічного відділення Send to hematology department